You are on page 1of 72

Total Marks : 200

TEST - 3(MOCK TEST - 3) Mark Scored : 89.33

1 Non-Resident Indians (NRIs) can invest in India as per a regulation of Foreign Exchange Management
(Transfer or issue of Security by a person resident outside India) Regulations, 2000. However, as per
the regulation
1. NRIs cannot invest in shares or debentures.
2. NRIs may contribute foreign capital either by way of capital contribution

Which of the above is/are correct?


A. 1 only
B. 2 only
C. Both 1 and 2
D. None

Your Answer : B
Correct Answer : B

Answer Justification :
2 77
Justification: Statement 1: As per Regulations, NRIs may invest in shares 3 2
or partly paid up shares
of an Indian company under Foreign Direct Investment (FDI) Scheme 1 5 0
subject to certain terms and
conditions. 9 4
-
m
Statement 2: NRIs may even acquire securities or unitslon
i .coa Stock Exchange in India on
a abroad) under the Portfolio Investment
repatriation basis (i.e. they can take the investment back
m
Scheme.
7 @g
NRIs even hold FCNR deposits in India i ra that earn them interest. We will cover FCNR in
sbanks
details in coming tests. k i tm
p ul
Q Source: DSM/SBS/KA a -(Release ID :169208)
is r
M
k it
2 Hambantotaldeep-sea port was recently see in news because
P u
A. Sri Lanka agreed to sell a strategic stake in this port to China.
B. A Major oil spill here was treated by genetically engineered bacteria.
C. India has signed a MoU with Seychelles to develop this port for defense related purposes.
D. It will be used as an alternate transit to the Straits of Malacca.

Your Answer : A
Correct Answer : A

Answer Justification :

Background: Hambantota port is a deep-water port in the southern tip of Sri Lanka.

It sits close to busy east-west shipping lanes connecting Europe and Asia.

Under the 99-year lease agreement, China will invest over a billion dollars in the port and marine-
related activities for developing this port.

www.insightsias.com 1
© Insights Active Learning | All rights reserved - 78168. You may not reproduce, distribute or exploit the contents in any form without
written permission by copyright owner. Copyright infringers may face civil and criminal liability
Total Marks : 200
TEST - 3(MOCK TEST - 3) Mark Scored : 89.33

Learning: The port is expected to play a key role in China’s Belt and Road initiative (BRI), which
will link ports and roads between China and Europe.

It is also considered that, this deal gives an advantage to China in the bunkering business, which
provides fuel to ships.

India is apprehensive that the port is part of Chinese ‘string of pearls’ with an objective to surround
India and dock its military vessels.

Q Source:
http://www.thehindu.com/news/international/sri-lanka-china-sign-11-bn-hambantota-port-deal/articl
e19385932.ece

3 A ‘Task Force on Shell Companies’ under the Joint Chairmanship of Revenue Secretary and Secretary,
Ministry of Corporate Affairs was constituted in February, 2017. What are ‘Shell Companies’?

7
A. Oil companies that cause heavy ecological damage
2 27
B. Fly by night operators
0 3
5
C. A corporate entity without active business operations
9 41
D. A company engaged in deep sea precious stones extraction-

. com
Your Answer : C
ai l
Correct Answer : C
gm
7 @
Answer Justification :
isra
it m
u k
Concept: Shell Company is a lcorporate entity without active business operations or significant
assets. - p
i sra
M
k
It can’t
l itbe asserted that shell corporations are illegal. They are deliberate financial
u
Parrangements by many big companies to avoid taxes without attracting legal actions.

Tax avoidance is not illegal, though it is not desirable.

Learning: But many shell companies park black money, carryout illegal transactions and
sometimes act as facilitators of money laundering.

Often, shell companies remain untraceable and happen to be the vehicle of choice for money
launderers, bribe givers and takers, tax evaders and financiers of terrorism.

Most of the shell companies are registered in tax havens like British Virgin Islands or Cayman
Islands.

Recently it was found that a single address hosted 75 firms and no employees! The taskforce

www.insightsias.com 2
© Insights Active Learning | All rights reserved - 78168. You may not reproduce, distribute or exploit the contents in any form without
written permission by copyright owner. Copyright infringers may face civil and criminal liability
Total Marks : 200
TEST - 3(MOCK TEST - 3) Mark Scored : 89.33

will look into such issues.

Q Source:
http://www.business-standard.com/article/economy-policy/kolkata-is-shell-companies-favourite-addr
ess-taxmen-plan-major-crackdown-117021300960_1.html

4 With reference to “flammable ice”, recently seen in news, consider the following statements:
1. It consists of methane trapped within water crystals.
2. It is another name for permafrost.
3. All of its reserves are trapped in plateaus at higher latitudes.
4. India is considering it as an alternative energy source.

Select the correct answer using the codes below.


A. 1, 3 and 4 only
B. 2 and 3 only 7
C. 1 and 4 only 2 27
D. only 0 3
5
41
Your Answer : -9
Correct Answer : A
. com
ai l
Answer Justification : gm
@
a7
r (also known as methane hydrate or methane clathrates)
sice
Justification: Statement 1: Flammable i
i
consists of methane trapped within
k tm
water crystals. It is the world’s largest natural gas resource
trapped beneath permafrostu l ocean sediment where low temperature and moderate pressure
and
p
combine to trap methane- in this specific way.
i sra
M
Statement 2: Permafrost appears on top of it. Permafrost is frozen chunks of ice that often contain
carbon and
it
lkvolatile gases.
Pu
Statement 3: China has successfully produced natural gas from methane hydrate, also known as
“flammable ice”, in an experimental project in the South China Sea (SCS).

Most of its reserves are located deep in ocean floors, especially continental shelves.

Statement 4: India, USA, China and Canada all are looking at it as an alternative energy source.

Learning: The methane hydrate is highly flammable and energy-intensive fuel as one cubic metre
of the compound can releases about 160 cubic metres of gas.

It can break down into water and methane after temperature is raised or pressure is lowered.

Q Source: China’s South China sea ocean drilling programme

5 The Government is implementing an Ethanol Blended Petrol (EBP) Programme in India. Consider the
following with reference to it.

www.insightsias.com 3
© Insights Active Learning | All rights reserved - 78168. You may not reproduce, distribute or exploit the contents in any form without
written permission by copyright owner. Copyright infringers may face civil and criminal liability
Total Marks : 200
TEST - 3(MOCK TEST - 3) Mark Scored : 89.33

1. The Government has introduced free market pricing mechanism for procurement of ethanol under
EBP Programme.
2. The Government has allowed procurement of ethanol produced from cellulosic and lignocellulosic
feedstock.

Which of the above is/are correct?


A. 1 only
B. 2 only
C. Both 1 and 2
D. None

Your Answer : C
Correct Answer : B

Answer Justification :

7 announced in
Background: For the development of Biofuels, a National Policy on Biofuels has7been
2 2
3 petrol with
the year 2009. So, under the EBP Oil Marketing Companies sell ethanol blended
0
percentage of ethanol up to 10% as per BIS specifications, depending1 5
upon its availability.
- 94
m the availability of ethanol can be
Justification: Statement 1: The prices are administered so that
o
improved by adequately incentivizing the producers. l.c
m ai
@g ethanol from bio feed stock.
Statement 2: This is an eco-friendly way to convert
7
isra of Biodiesel in the country, the Government had also
Learning: In order to encourage production
announced the Biodiesel Purchase i m in October, 2005.
tPolicy
ul k
p
The Government has also- permitted the direct sale of Biodiesel (B-100) for blending with High
Speed Diesel to allisra
consumers, in accordance with the specified blending limits and the standards
M
l k it
specified by the BIS.

Pu RM/RS- USQ1844 - LS (Release ID :169151)


Q Source:

6 With reference to Indian Green Building Council (IGBC), consider the following statements.
1. It is a body under Department of Urban Development.
2. It organizes the Green Building Congress.
3. It offers green building rating programmes.

Select the correct answer using the codes below.


A. 1 only
B. 2 and 3 only
C. 2 only
D. 3 only

Your Answer : D
Correct Answer : B

www.insightsias.com 4
© Insights Active Learning | All rights reserved - 78168. You may not reproduce, distribute or exploit the contents in any form without
written permission by copyright owner. Copyright infringers may face civil and criminal liability
Total Marks : 200
TEST - 3(MOCK TEST - 3) Mark Scored : 89.33

Answer Justification :

Justification: The IGBC is part of the Confederation of Indian Industry (CII) formed in 2001.

It offers a wide array of services including developing new green building rating programmes,
green building training programmes and certification services.

It also organises Green Building Congress, its annual flagship event on green buildings.

It closely works with several State Governments, Central Government, World Green Building
Council etc.

Q Source: IGBC conference 2017


7
2 27
7 Consider the following about Advance Pricing Agreements (APA) scheme. 0 3
1. It was introduced by the Income-tax (IT) Act. 4 15
2. It cannot be signed with individual companies. -9
3. It endeavors to provide certainty to taxpayers in the domain
. com
of transfer pricing.
4. It is not applicable to international transactions. i l a
m
@g
s r a7
Select the correct answer using the codes below.
A. 1 and 3 only
m i
t
lki
B. 2 and 4 only
C. 2, 3 and 4 only u
D. 1, 3 and 4 only -p
i sra
Your Answer t: A M
lki : A
CorrectuAnswer
P
Answer Justification :

Concept: Suppose two subsidiaries A and B of the same company X are located in two different tax
regimes. Say A is subjected to more taxes, and B has to pay less tax.

If company X transfers most of its profit from A to B, it will need to less taxes then, because B is
taxed at lower rate.

This is called transfer pricing, when profits are transferred to subsidiaries to reduce tax burden.

Justification: Statement 2 and 3: It is signed with companies so that this tax evasion can be
regulated. It also gives certainty to the tax department in terms of revenue collection.

Q Source: The Central Board of Direct Taxes (CBDT) has entered into 9 Unilateral Advance Pricing
Agreements (UAPAs) with Indian taxpayers in July, 2017.

www.insightsias.com 5
© Insights Active Learning | All rights reserved - 78168. You may not reproduce, distribute or exploit the contents in any form without
written permission by copyright owner. Copyright infringers may face civil and criminal liability
Total Marks : 200
TEST - 3(MOCK TEST - 3) Mark Scored : 89.33

8 Consider the following about Jiyo Parsi scheme.


1. It is a Central Sector Scheme focused on increasing employment opportunities for Parsis in India.
2. It provides central assistance for expatriate Parsis to transfer their skills to native Parsis.

Which of the above is/are correct?


A. 1 only
B. 2 only
C. Both 1 and 2
D. None

Your Answer : D
Correct Answer : D

Answer Justification :

Justification: Its main objective is to reverse the declining trend of Parsi population by adopting a
7
scientific protocol and structured interventions. 27
0 32
It has two components: Medical Assistance and Advocacy (Counselling).
4 15The first phase of the
scheme was launched initiated in 2013.
-9
. com
Learning: Minorites are defined both at the Central and
ai l State level.
m
g welfare is the responsibility of the Central
Parsis form a minority community in India, and their
7 @
sra
government.
i
i
For a state declared minority, state
k tm
governments provide for welfare plans.
l
- pu of Minority Affairs has launched Jiyo Parsi Publicity Phase-2 in
Q Source: The Union Ministry
Mumbai i sra
k itM
9 ‘Mission P ul
Purpose’, ‘Breakthrough Starshot’ and ‘Sprites’ were recently seen in news in context of

A. Interstellar missions
B. Cost effective sea water desalination plants
C. Autonomous self-driving cars
D. Genetically engineered clonal species

Your Answer :
Correct Answer : A

Answer Justification :

Learning: Starshot is a comprehensive space program launched under the US $100 million
Breakthrough Initiatives, announced by Yuri Milner and Stephen Hawking to develop and launch
practical interstellar space missions.

www.insightsias.com 6
© Insights Active Learning | All rights reserved - 78168. You may not reproduce, distribute or exploit the contents in any form without
written permission by copyright owner. Copyright infringers may face civil and criminal liability
Total Marks : 200
TEST - 3(MOCK TEST - 3) Mark Scored : 89.33

The program aims to demonstrate proof of concept for light-propelled spacecraft that could fly
at 20 per cent of light speed.

Its main objective is to send one-gram chips to star systems beyond the solar system in search
of extraterrestrial intelligence.

Mission Purpose was designed to test the performance of the ‘Sprites’ (tiniest satellite) electronics
when in orbit.

It will also explore the concept of solar sail propulsion in which spacecraft can be powered
using only the sun’s radiation.

Sprites have been developed by researchers at Cornell University. Each of the mini Sprite
77 each.
spacecrafts are built on a single 3.5*3.5 cm circuit board. They weigh just four grams
2
2
5 03
41 miniaturization that can
These tiny satellites also mark the next step in the field of spacecraft
9
contribute to the development of centimeter and gram-scale - StarChips envisioned under the
o m
Breakthrough Starshot project.
il .c
m a
Q Source: Often in news @ g
7
isra
it
10 Muntra, recently rolled out by thekDefense
m
Research and Development Organization (DRDO) is a/an
u l
-p
i sra operated tank
A. Unmanned remotely
M mine
B. Stealth land
t
k i
C. Indigenously
l developed fifth generation aircraft
D. P u
Airborne Radar system for fighter planes

Your Answer : A
Correct Answer : A

Answer Justification :

Learning: It is India’s first unmanned, remotely operated tank at its Chennai lab.

It can be used for surveillance, mine detection and reconnaissance in areas with nuclear and
biological threats.

It is also likely to be used in Naxal-hit areas.

Q Source: Defense related updates

11 ‘Sagar Vani’ system released by the Union Ministry of Earth Science is

www.insightsias.com 7
© Insights Active Learning | All rights reserved - 78168. You may not reproduce, distribute or exploit the contents in any form without
written permission by copyright owner. Copyright infringers may face civil and criminal liability
Total Marks : 200
TEST - 3(MOCK TEST - 3) Mark Scored : 89.33

A. An integrated information dissemination system that will serve the coastal community
regarding their safety
B. A SONAR system for deep sea mineral exploration
C. A language code for Indian Navy to communicate more securely
D. None of the above

Your Answer : A
Correct Answer : A

Answer Justification :

Learning: It serves the coastal community, especially the fishermen community with the advisories
and alerts towards their livelihood as well as their safety at Sea.

It is a software platform where various dissemination modes will be integrated on a single


central server. 7
2 27
0 3
5 (Facebook, Twitter,
1Media
4
It includes Multi Lingual SMS, Voice Call/Audio Advisory, Social
etc.) etc. -9
. com
ai l
g m
Application: It can disseminate services in local languages using advanced artificial intelligence
(AI) and machine learning capabilities. @ 7
isra
This ‘Sagar Vani’ system k it m
compares with the most advanced countries’ services in terms of
u l
p
speed of delivery, diverseness
-
of services and omni channel capabilities.
a
isr
k it Malso has facility to provide access to various stakeholders (State Fishery
The system
ul
Departments,
Pfurther
Disaster Management Authorities, NGOs etc.) so that they will be able to
disseminate these ocean information and alerts to the user community.

Q Source: Initiative of Ministry of Earth Sciences: PIB

12 With reference to the World Monuments Fund (WMF), consider the following:
1. It is a dedicated fund under UNESCO managed by the affiliates of UNESCO.
2. Every member country of UNESCO has agreed to contribute to this fund.
3. Every year, WMF presents the Hadrian Award to international leaders who have advanced the
preservation of world architecture.

Select the correct answer using the codes below.


A. 1 only
B. 3 only
C. 2 and 3 only
D. 2 only

www.insightsias.com 8
© Insights Active Learning | All rights reserved - 78168. You may not reproduce, distribute or exploit the contents in any form without
written permission by copyright owner. Copyright infringers may face civil and criminal liability
Total Marks : 200
TEST - 3(MOCK TEST - 3) Mark Scored : 89.33

Your Answer :
Correct Answer : B

Answer Justification :

Justification: Statement 1: WMF is a private, international, non-profit organization dedicated to


the preservation of historic architecture and cultural heritage sites around the world.

Statement 2: Founded in 1965, through donations and matching funds, WMF has worked with local
community and government partners worldwide to safeguard and conserve places of historic value
for future generations.

To date, WMF has worked in more than 91 countries, including many UNESCO World Heritage
Sites.

Statement 3: This applies both for preservation of world art and architecture.
7
Q Source: Additional Research:
2 27
03
http://www.thehindu.com/news/international/jesuss-tomb-to-be-unveiled-to-public/article17566017.e
5
ce 1 4
-9
m
13 With reference to the technology, SurfNet, that was recently
i l .coseen in news, consider the following:
1. It is an artificial intelligence (AI) based software. ma
2. It can construct 3D models from 2D shapes. @g
7
ra and organelles of human beings.
3. It is presently being used for printing tissues
is
it m
lkthe codes below.
Select the correct answer using
u
A. 1 and 2 only - p
B. 2 and 3 only sra
C. 3 only M
i
t
lki
D. 2 only
Pu
Your Answer :
Correct Answer : A

Answer Justification :

Justification: Statement 1: SurfNet software utilizes machine learning to analyze 2D shapes and
convert them into projected 3D forms.

In an advance version, it can allow future robots to navigate in the real world.

In addition to transforming 2D shapes, the technology can also merge two shapes into one
another.

Statement 3: They were already being printed. Also, this is only a software, not a 3D printer.

www.insightsias.com 9
© Insights Active Learning | All rights reserved - 78168. You may not reproduce, distribute or exploit the contents in any form without
written permission by copyright owner. Copyright infringers may face civil and criminal liability
Total Marks : 200
TEST - 3(MOCK TEST - 3) Mark Scored : 89.33

Learning (applications): In future it can even be used in object recognition and self-driving cars.
SurfNet will give them ability to understand the 3D environment around them using standard 2D
cameras.

It can also be used to create 3D content for virtual reality (VR) and augmented reality (AR) by
simply using standard 2D photos.

Q Source: Breakthrough initiative has been in news for a long time

14 Who administers the oath of the President of India?

A. Vice-President
B. Prime Minister of India
C. Speaker, Lok Sabha
D. Chief Justice of India
7
Your Answer : D 2 27
Correct Answer : D 0 3
5
41
Answer Justification : -9
. com
Learning: According to Article 60 of the Constitution l
aiof India, every President and every person
m
acting as President or discharging the functionsgof the President before entering upon office shall
make and subscribe oath to the office.
a 7@
is r
m
itthe oath to preserve, protect and defend the constitution.
l k
The oath inter alia includes
- pu
i ra
sPresident
In this sense, takes oath as guardian of Indian Constitution.
t M
u lki
PThe oath is administered by Chief Justice of India or in his absence by the senior-most Judge
of the Supreme Court available.

Q Source: Presidential Oath: July 2017

15 The Petrapole-Benapole Integrated Check Post is a major route for

A. India-Bangladesh bilateral trade


B. India-Bhutan border trade
C. India-China transit trade
D. India-Pakistan Wagha border trade

Your Answer : A
Correct Answer : A

www.insightsias.com 10
© Insights Active Learning | All rights reserved - 78168. You may not reproduce, distribute or exploit the contents in any form without
written permission by copyright owner. Copyright infringers may face civil and criminal liability
Total Marks : 200
TEST - 3(MOCK TEST - 3) Mark Scored : 89.33

Answer Justification :

Learning: In order to facilitate movement of cargo across the border, India and Bangladesh have
recently agreed to operate Petrapole-Benapole ICP 24x7.

The 24x7 operationalization of Petrapole-Benapole ICP is expected to be a significant


milestone towards expeditious clearance of cargo and, hence boost the bilateral trade
between the two countries.

Land Ports Authority of India (LPAI) and Central Board of Excise and Customs (CBEC) are the
implementing agencies for this agreement from the Indian side.

Q Source: MJPS (Release ID :169332)

16 With reference to the Unorganised Workers' Social Security Act 2008, consider2the 2 following: 77
3
1. It covers self-employed workers as well as daily-wage workers.
1 50
2. It provides for constitution of National Social Security Board.
- 94
3. It issues a universal labour identification number (ULIN) to each labour in the unorganized sector.
. com
Select the correct answer using the codes below. ai l
A. 1 and 2 only g m
B. 3 only a 7@
is r
C. 2 and 3 only
m
D. 1 only kitu l
Your Answer : A a -p
isr
Correct Answer : A
k itM
l
AnsweruJustification :
P
Justification: Statement 1: It is for the welfare of the unorganised workers (meaning home-based
workers, self-employed workers or daily-wage workers).

Statement 2: The National Social Security Board at the Central level shall recommend formulation
of social security schemes viz life and disability cover, health and maternity benefits, old age
protection and any other benefit as may be determined by the Government for unorganised
workers.

Statement 3: LIN has been recently issued under Shramev Jayate Yojana since 2015. It wasn’t under
this act.

Q Source: AK (Release ID :169273)

17 With reference to manufacture of firecrackers in India, consider the following:


1. It is mandatory for firecracker manufacturers to mention the composition of explosives and their

www.insightsias.com 11
© Insights Active Learning | All rights reserved - 78168. You may not reproduce, distribute or exploit the contents in any form without
written permission by copyright owner. Copyright infringers may face civil and criminal liability
Total Marks : 200
TEST - 3(MOCK TEST - 3) Mark Scored : 89.33

quantity on the packet.


2. It is not mandatory to mention the consequential effects of fireworks on the environment and
human health on the label of the packet.

Which of the above is/are correct?


A. 1 only
B. 2 only
C. Both 1 and 2
D. None

Your Answer : B
Correct Answer : C

Answer Justification :

Justification: Statement 1: This is as per the Explosives Rules, 2008.


77
3 22of fireworks on the
Statement 2: However, it is not mandatory to mention the consequential effects
environment and human health on the label of the packet or carton. 50 1
4
-9
This may be because the effects are well known or c
. omthese products aren’t sold like general
that
consumer products. ai l
gm
7 @
Their sale can be regulated by the a
srdistrict administration (for e.g. by providing an isolated
i
tm
ground for sale of firecrackers).
i
lk
- pu
ra ID :169309)
Q Source: MJPS (Release
is
M
l k it
Pu
18 As per extant FDI policy in India
1. Foreign investment up to 100 percent is permitted for a company engaged in the business of
manufacturing mobile phones.
2. There is a 30 per cent local sourcing of component for any FDI related manufacturing activity
undertaken in India.

Which of the above is/are correct?


A. 1 only
B. 2 only
C. Both 1 and 2
D. None

Your Answer : A
Correct Answer : A

Answer Justification :

www.insightsias.com 12
© Insights Active Learning | All rights reserved - 78168. You may not reproduce, distribute or exploit the contents in any form without
written permission by copyright owner. Copyright infringers may face civil and criminal liability
Total Marks : 200
TEST - 3(MOCK TEST - 3) Mark Scored : 89.33

Justification: Statement 1: This is under the automatic route where investment in the country can
be made without prior approval either of the Government or the Reserve Bank of India.

FDI is largely a matter of private business decisions, and therefore, no such details regarding
efforts made by foreign companies to set up for e.g. Apple i-phone facilities are required.

Statement 2: This isn’t applicable to all manufacturing units, for e.g. it is a restriction for mobile
phone manufacturing units in India.

Recently Apple submitted a representation to waive off this requirement because it would
affect their profit margins.

However, in case of ‘state-of-art’ and ‘cutting-edge technology’, sourcing norms can be


relaxed subject to Government approval. 77 2
2
5 03
1 the opening of first store,
This relaxation will be valid for an initial period of three years4from
- 9
and thereafter, such entities will be required to meet the domestic sourcing norms.

. com
ai l
Q Source: MJPS (Release ID :169307)
gm
@
s r a7
19 Archaeological Sites are often preservedi and cleaned using several chemical conservation
approaches by ASI. Which of these k i tm under such approaches?
come
l
ulichens
1. Using a mixture of moss and
- p on monuments

i sra
2. Adding Suspended Particulate Matter (SPM) to clean art pieces
3. Removal of calcareous deposits and accretions
k itM
4. Bio-cide treatment
l
Pu
Select the correct answer using the codes below.
A. 1, 2 and 3 only
B. 3 and 4 only
C. 2, 3 and 4 only
D. 1 and 4 only

Your Answer :
Correct Answer : B

Answer Justification :

Justification: Statement 1: There are certain biological factors such as Growth of Moss, Lichen,
Algae, Fungi and higher plants on a monument.

They not only impart a ugly look to the monument but also cause physical & chemical damaged to
the building material. So, 1 is incorrect.

www.insightsias.com 13
© Insights Active Learning | All rights reserved - 78168. You may not reproduce, distribute or exploit the contents in any form without
written permission by copyright owner. Copyright infringers may face civil and criminal liability
Total Marks : 200
TEST - 3(MOCK TEST - 3) Mark Scored : 89.33

Statement 2: Suspended Particulate Matter (SPM) and other chemically active pollutant species
along with dust results in disfiguring of the monument.

Similarly, variation in climatic conditions, moisture and intense solar radiation are also responsible
for the decay of specific building materials.

Statement 3 and 4: General cleaning of monuments, mud pack cleaning, removal of calcareous
deposits and accretions, bio-cide treatment, consolidation and water repellent treatment, are some
of the known ways of chemical treatment.

Learning: Ancient Monuments and Archaeological Sites and Remains Act – 2010 provides for the
preservation of ancient and historical monuments, archaeological sites, and remains of national
importance and also provides penalty against acts of human vandalism.

Other methods of cleaning: Mud Pack Cleaning is used for the cleaning of plain and decorated
marble surface.
7
2 27
3 and other marble
This method is being used successfully for the conservation of Taj Mahal
0
5
41
structures.
- 9
m
o detergent is used for the easy
Very dilute mixture of ammonia solution and a non.c ionic
i l
a mild brushing.
removal of accretions and biological growth with
m
7 @g
i
Bleaching powder slurry in aqueoussramedium is used to remove micro-vegetation from lime
plastered surface.
k i tm
p ul
a -
isr
M
l k it
Pu

www.insightsias.com 14
© Insights Active Learning | All rights reserved - 78168. You may not reproduce, distribute or exploit the contents in any form without
written permission by copyright owner. Copyright infringers may face civil and criminal liability
Total Marks : 200
TEST - 3(MOCK TEST - 3) Mark Scored : 89.33

7
2 27
0 3
Q Source: http://www.pib.nic.in/newsite/mbErel.aspx?relid=105157 5
41
-9
20 Consider the following about India Brand Equity Foundation o m and associated provisions.
(IBEF)
i l
1. It is a non-profit NGO certifying khadi and handloomaproducts.
. c
m
g awareness of the ‘Made in India’ label in
2. Its objective is to promote and create international
@
markets overseas.
s r a7 in 2015 emphasises the need of a branding
3. India’s Foreign Trade Policy Statement
m i released
strategy. i t
lkplantations,
4. Activities of IBEF are limiteduto textiles and leather sectors.
- p
a
sr using the codes below.
Select the correctianswer
M
A. 2 and i3tonly
B. 1u k
l 4 only
and
P
C. 1, 2 and 4 only
D. 1, 2 and 3 only

Your Answer :
Correct Answer : A

Answer Justification :

Justification: Statement 1: It is a trust established by the Department of Commerce, Ministry of


Commerce and Industry.

Statement 2: IBEF's primary objective is to promote and create international awareness of the
Made in India label in markets overseas and to facilitate dissemination of knowledge of Indian
products and services. Towards this objective, IBEF works closely with stakeholders across
government and industry

Statement 3: This is to promote Indian exports as well as push Make in India campaign.

www.insightsias.com 15
© Insights Active Learning | All rights reserved - 78168. You may not reproduce, distribute or exploit the contents in any form without
written permission by copyright owner. Copyright infringers may face civil and criminal liability
Total Marks : 200
TEST - 3(MOCK TEST - 3) Mark Scored : 89.33

Statement 4: IBEF has undertaken focused branding activities for sectors namely engineering,
pharma, plantations (tea, coffee and spices), services, textiles and leather.

Q Source: MJPS (Release ID :169316)

21 The Government has launched a new scheme namely “Financial Support to MSMEs in ZED
Certification Scheme”. The objective(s) of the scheme includes
1. Promote adaptation of energy efficient manufacturing
2. Drive manufacturing with adoption of Zero Defect production processes and without impacting the
environment.
3. Establish a MSME Product Certifying Authority (MPCA) to assess quality of their products
4. Support ‘Make in India’ campaign.

Select the correct answer using the codes below.


A. 1 and 3 only
B. 2 and 4 only 7
C. 1, 2 and 4 only 2 27
D. 1, 2, 3 and 4 0 3
5
41
Your Answer : -9
Correct Answer : C
. com
ai l
Answer Justification : gm
@
sr a7
Justification: ZED strives for quality imanufacturing.

k i tm
The objective of the scheme u l promotion of ZED manufacturing amongst MSMEs and ZED
for
- p
Assessment for their certification so as to (these are other than what is mentioned in the question):
sr a
M i
k
Developit an Ecosystem for Zero Defect Manufacturing in MSMEs.
ul
P
Encourage MSMEs to constantly upgrade their quality standards in products and processes.

Develop professionals in the area of ZED manufacturing and certification.

Q Source: AK/RM (Release ID :169325)

22 Ministry of Tourism has released Guidelines for promotion of Medical and Wellness Tourism as Niche
Tourism Product. As per the guidelines
1. Ministry of Tourism offers financial support to accredited Medical and Wellness Tourism Service
Providers
2. Medical tourists from any country shall be provided Visa on arrival and e-visa facilities

Which of the above is/are correct?

www.insightsias.com 16
© Insights Active Learning | All rights reserved - 78168. You may not reproduce, distribute or exploit the contents in any form without
written permission by copyright owner. Copyright infringers may face civil and criminal liability
Total Marks : 200
TEST - 3(MOCK TEST - 3) Mark Scored : 89.33

A. 1 only
B. 2 only
C. Both 1 and 2
D. None

Your Answer :
Correct Answer : A

Answer Justification :

Justification: Tourism products including medical and wellness tourism are mostly driven by the
private sector and cost of such services are determined by market forces.

However, the Ministry of Tourism is taking steps to promote this from the side of government
as well.
7
2 27
03
The grant is for Marketing Development Assistance, for Publicity, organising
5
workshop/events/seminars and for organising Wellness and Medical
9 41 Tourism Promotion
shows. -
m
i l .co
Q Source: Sanjay Kumar/jk/Parl. No. 1/26-07-2017: m a(Release ID :169061)
7 @g
sra and Engineering Research Board (SERB) to
23 The VAJRA scheme was launched by theiScience
k i tm
p ul and research and development (R&D) systems to the best of
A. Connect the Indian academic
a
global science rand
-scientists
M is
B. Develop anti-neurotoxins that can be used to neutralize agents used in a potential chemical
it
ulk mobile air-defence guards to marine ships used as warships
warfare
C. P
Provide
D. Increase participation from local ground level innovations to mainstream science and
technology

Your Answer :
Correct Answer : A

Answer Justification :

Learning: SERB is a statutory body of the Department of Science and Technology (DST).

It recently launched a ‘Visiting Advanced Joint Research (VAJRA) Faculty Scheme’ for a
sustained international collaborative research.

The scheme offers adjunct / visiting faculty assignments to overseas scientists, faculty
members and R&D professionals including NRI and OCI to undertake high quality

www.insightsias.com 17
© Insights Active Learning | All rights reserved - 78168. You may not reproduce, distribute or exploit the contents in any form without
written permission by copyright owner. Copyright infringers may face civil and criminal liability
Total Marks : 200
TEST - 3(MOCK TEST - 3) Mark Scored : 89.33

collaborative research in cutting edge areas of science and technology.

This is to be done with one or more Indian collaborators of public funded academic and
research institutions of India.

Q Source: RDS/nb (Release ID :169081)

24 What is the importance of Kanchi in context of Ancient India?


1. Ghatika at Kanchi was an ancient centre of leaning attracting students from India and abroad.
2. Founder of the Kadamba dynasty, Mayurasarman, studied Vedas at Kanchi.
3. Dharmapala, who later became the Head of the Ancient Nalanda University, belonged to Kanchi.
4. It was known as the religious capital of the Southern India.
5. Kanchi hosts Kailasanatha which is one of the largest and most ornate ancient temples in the whole
of India.
7
2 27
Select the correct answer using the codes below.
0 3
5
A. 2 and 5 only
41
B. 1 and 3 only -9
C. 3 and 4 only
. com
D. 1, 2, 3, 4 and 5
ai l
gm
Your Answer : 7 @
Correct Answer : D
isra
i t m
Answer Justification :
p ulk
a -
Justification: Once
i sra capital of the Pallava dynasty, Kanchipuram was also a noted centre of
M and Sanskrit scholars. Dinganaga, a Buddhist writer came to study at Kanchi.
learning for Tamil
l k it
Pu
It hosts many temples dedicated to Shiva and Vishnu.

Kanchi was also the home of the famous 6th century CE poet Bharavi who wrote the
Kiratarjuniya and the famous 11th to 12th century CE Hindu philosopher Ramanuja.

Still today an important religious centre, the site has over hundred temples and is also noted
for its production of fine silk saris.

Q Source: Additional Research: Page 118: TN 11th Standard Textbook

25 Pancha Siddhantika, a work of Varahamihira, deals with

A. Geographical domains of India

www.insightsias.com 18
© Insights Active Learning | All rights reserved - 78168. You may not reproduce, distribute or exploit the contents in any form without
written permission by copyright owner. Copyright infringers may face civil and criminal liability
Total Marks : 200
TEST - 3(MOCK TEST - 3) Mark Scored : 89.33

B. Astronomical systems
C. Occult and mysticism
D. Principles of social living

Your Answer : B
Correct Answer : B

Answer Justification :

Learning: He was also a great authority on astrology. In this context, he composed Pancha
Siddhantika, about the five astronomical systems. Along with Aryabhatta, his contribution is
considered seminal in Ancient India.

His work Brihadsamhita is a great work in Sanskrit literature. It deals with a variety of subjects like
astronomy, astrology, geography, architecture, weather, animals, marriage and omens.

His Brihadjataka is considered to be a standard work on astrology. 7


2 27
Q Source: Page 108: TN 11th Standard Textbook 0 3
5
41
-9
o m
26 In India, the safety aspects of genetically modified crops are assessed by which of these bodies?
1. Institutional Biosafety Committees (IBSCs) il. c
2. Review Committee on Genetic Manipulation (RCGM) m a
g
3. Genetic Engineering Appraisal Committee (GEAC)
a 7@
4. National Biopiracy authority (NBA) s r i
m
kit
ul the codes below.
Select the correct answer using
p
A. 1, 2 and 3 only
B. 2 and 4 onlyisr
a-
M
C. 1 and 3tonly
D. 2, 3lk
i
and 4 only
Pu
Your Answer : A
Correct Answer : A

Answer Justification :

Background: GEAC is the apex body constituted in the Ministry of Environment and Forests.

It was set up under 'Rules for Manufacture, Use, Import, Export and Storage of Hazardous
Microorganisms/Genetically Engineered Organisms or Cells 1989', under the Environment
Protection Act, 1986.

The Rules of 1989 also define five competent authorities i.e. IBSC, RCGM, GEAC, State
Biotechnology Coordination Committee (SBCC) and District Level Committee (DLC) for
handling of various aspects of the rules.

www.insightsias.com 19
© Insights Active Learning | All rights reserved - 78168. You may not reproduce, distribute or exploit the contents in any form without
written permission by copyright owner. Copyright infringers may face civil and criminal liability
Total Marks : 200
TEST - 3(MOCK TEST - 3) Mark Scored : 89.33

Justification: Statement 1: Its main function is to note, examine and approve proposals involving r-
DNA work; to ensure adherence of r-DNA Safety Guidelines- 1990 of Government and inspection of
containment facilities at R&D and production units.

Statement 2: Main functions of RGCM are:

To bring out manuals of guidelines specifying producers for regulatory process on GMOs in
research, use and applications including in industry with a view to ensure environmental
safety.

To review all on going r-DNA projects involving high risk category and controlled field
experiments

To lay down producers for restriction or prohibition, production, sale, import & use of GMOs
both for research and applications. 77 2
2
5 03
41research purposes.
To authorize imports of GMOs/ transgenes/ transgenic seeds for
-9
. com
Q Source: RDS/nb (Release ID :169320)
ai l
gm
7 @
ra the Pattini cult in Tamilnadu?
27 With reference to Ancient India, who introduced
is
it m
A. Cheran Senguttuvan
u lk
B. Rajaraja II - p
C. Pulakesin I sra
D. Mi
Narasinghvarmana
l k it
Your P u :
Answer
Correct Answer : A

Answer Justification :

Justification: He belonged to 2nd century A.D.

Senguttuvan introduced the Pattini cult or the worship of Kannagi as the ideal wife in Tamil
Nadu.

The stone for making the idol of Kannagi was brought by him after his Himalayan expedition.

The consecration ceremony was attended by many princes including Gajabhagu II from Sri
Lanka.

www.insightsias.com 20
© Insights Active Learning | All rights reserved - 78168. You may not reproduce, distribute or exploit the contents in any form without
written permission by copyright owner. Copyright infringers may face civil and criminal liability
Total Marks : 200
TEST - 3(MOCK TEST - 3) Mark Scored : 89.33

Learning: His younger brother was Elango Adigal, the author of Silappathigaram.

Among his military achievements, his expedition to the Himalayas was remarkable. He defeated
many north Indian monarchs.

Q Source: Page 87: TN 11th Standard Textbook

28 Family Participatory Care (FPC) as an important concept of health care mainly focuses on

A. Newborn healthcare
B. Geriatric population
C. Urban sanitation and hygiene
D. Health insurance in India

Your Answer : A
Correct Answer : A 7
2 27
Answer Justification : 0 3
5
41
9 nurturing in order to survive
Learning: Sick and newborn are highly vulnerable and require -careful
the neonatal period and first year of life. om
a il.c
This approach provides for partnership @
gm
between health care staff and families in care of sick
r a7 Care Units (SNCU).
newborns admitted in the SpecialsNewborn
m i
t
u lki
p of parents-attendants are built in newborn care through a structured
Under it, the capacities
-
sra
training programme.
i
k itM
l has recently released Operational Guidelines on FPC.
Pu
MoHFW

The guidelines will be shared with the states for implementation to further improve the
quality of care provided in the SNCUs across the country.

Q Source: http://pib.nic.in/newsite/PrintRelease.aspx?relid=169187

29 The NITI Aayog has approved several high-tech mas rapid transport proposals to improve public
transport. Consider the following about them.
1. Hyperloop is a high air pressure tube connecting cities at speeds matching that of an aircraft.
2. Freight rail roads will be elevated corridors with rail lines where freight trucks can move on rails at
high speed.
3. Pod taxis are underground transit systems that run entirely on solar power.
4. Hybrid electric buses combine a conventional internal combustion engine with an electric
propulsion system.

www.insightsias.com 21
© Insights Active Learning | All rights reserved - 78168. You may not reproduce, distribute or exploit the contents in any form without
written permission by copyright owner. Copyright infringers may face civil and criminal liability
Total Marks : 200
TEST - 3(MOCK TEST - 3) Mark Scored : 89.33

Select the correct answer using the codes below.


A. 1 and 3 only
B. 2 and 4 only
C. 2, 3 and 4 only
D. 1 and 2 only

Your Answer : A
Correct Answer : B

Answer Justification :

Justification: This proposal includes high-tech mass rapid transportation technologies such as
metrino, hyperloop, pod taxis, stadler buses, hybrid buses and freight rail road.

Statement 1: They are near-vacuum tubes so that air friction in travel can be reduced.

Statement 2: A dedicated corridor allows for greater efficiency and speed. 7


2 27
Statement 3: These are usually cabled taxis running on an elevated drive 0
train. 3
1 5
-
Statement 4: The addition of a battery-powered electric motor increases 94
the fuel efficiency of
hybrids in a number of ways. o m
a il.c
gm
7 @
Idle-off feature turns off the car's conventional engine when the vehicle is stopped, saving
fuel.
sr a
i
k i tm
ul for the air conditioner and accessories while the vehicle idles at
The battery providespenergy
a - and the electric motor can start the vehicle moving again.
r
stoplights or in traffic,
is
t M
lki the conventional engine will reengage to provide more power for acceleration.
Ifuneeded,
P
Learning: "Regenerative braking" is another fuel-saving feature. Conventional cars rely entirely on
friction brakes to slow down, dissipating the vehicle's kinetic energy as heat.

Regenerative braking allows some of that energy to be captured, turned into electricity, and
stored in the batteries.

This stored electricity can later be used to run the motor and accelerate the vehicle.

Q Source:
http://www.livemint.com/Politics/8fnjxsUr13noU98kvkPerO/Transport-ministry-to-explore-6-new-ma
ss-rapid-transportatio.html

www.insightsias.com 22
© Insights Active Learning | All rights reserved - 78168. You may not reproduce, distribute or exploit the contents in any form without
written permission by copyright owner. Copyright infringers may face civil and criminal liability
Total Marks : 200
TEST - 3(MOCK TEST - 3) Mark Scored : 89.33

30 Aadhaar is presently being used as an identifier in respect of following school education related
schemes?
1. Mid-Day Meal Scheme (MDMS)
2. Sarva Shiksha Abhiyan (SSA)
3. Rashtriya Madhyamik Shiksha Abhiyan (RMSA)
4. Inclusive Education for Disabled at Secondary Stage (IEDSS)

Select the correct answer using the codes below.


A. 1 and 2 only
B. 2, 3 and 4 only
C. 3 and 4 only
D. 1, 2, 3 and 4

Your Answer :
Correct Answer : D

7
Answer Justification :
2 27
0 3
15 of provisions.
Justification: This is to reduce ghost beneficiaries, leakages in distribution
- 94
min the governance of the schemes.
Also, this enforces accountability, transparency and efficiency
o
il. c
Other covered schemes are:
m a
g
a 7@
r
s for Secondary Education (NSIGSE)
National Scheme of Incentive toiGirls
it m
l k
- pu Scholarship Scheme (NMMSS)
National Means-cum-Merit
i sra
k itM
Centrally Sponsored Scheme for providing quality education in Madrasa (SPQEM)
u l
P
Q Source: GG/AK/RK/Aadhaar for Social Welfare: (Release ID :169280)

31 he Satavahanas patronized Buddhism and Brahmanism. The evidence of this is clear from the fact
that
1. They built several chaityas and viharas.
2. They prohibited Buddhist monks from acquiring land to help in their spiritual pursuit.
3. They persecuted Brahmanism.
4. They patronized the Prakrit language and literature.

Select the correct answer using the codes below.


A. 1, 2 and 3 only
B. 1 and 4 only
C. 1, 3 and 4 only
D. 2 and 3 only

www.insightsias.com 23
© Insights Active Learning | All rights reserved - 78168. You may not reproduce, distribute or exploit the contents in any form without
written permission by copyright owner. Copyright infringers may face civil and criminal liability
Total Marks : 200
TEST - 3(MOCK TEST - 3) Mark Scored : 89.33

Your Answer : B
Correct Answer : B

Answer Justification :

Justification: Statement 1: Vashishtaputra Pulamayi repaired the old Amaravathi stupa. Their
architecture in Nagarjunakonda was also notable.

Statement 2: They also made grants of villages and lands to Buddhist monks.

Statement 3: Brahmanism was revived by the Satavahanas along with the performance of
asvamedha and rajasuya sacrifices.

Statement 4: Hala’s Sattasai is an excellent piece of Prakrit literature.

Q Source: Page 77: TN 11th Standard Textbook

277
2
03
32 With reference to Ancient India, in the northern part of the country, the village headman was known
as the 5
9 41
-
A. Gramabhojaka m
B. Agrahara
i l .co
C. Dhammsurna m a
D. Sentokwara
7 @g
is ra
Your Answer : A
k i tm
ul
Correct Answer : A
- p
Answer Justificationr a:
M is
l it
Learning:kUsually, men from the same family held the position for generations or via hereditary.
P u
The grama bhojaka was often the largest landowner. Generally, he had slaves and hired
workers to cultivate the land.

Besides, as he was powerful, the king often used him to collect taxes from the village.

He also functioned as a judge, and sometimes as a policeman.

Apart from the gramabhojaka, there were other independent farmers, known as grihapatis,
most of whom were smaller landowners.

Q Source: Revision Previous tests: Chapter 7: 6th NCERT History

www.insightsias.com 24
© Insights Active Learning | All rights reserved - 78168. You may not reproduce, distribute or exploit the contents in any form without
written permission by copyright owner. Copyright infringers may face civil and criminal liability
Total Marks : 200
TEST - 3(MOCK TEST - 3) Mark Scored : 89.33

33 Sangam poems mention the term muvendar. It was used for

A. Land allotted to temples by Kings


B. Administrative arrangements made to felicitate scholars of Tamil
C. Heads of three ruling dynasties of Southern India
D. Classical commentary on Vedas

Your Answer : A
Correct Answer : C

Answer Justification :

Learning: This is a Tamil word meaning three chiefs, used for the heads of three ruling families,
the Cholas, Cheras, and Pandyas, who became powerful in south India around 2300 years ago.

Each of the three chiefs had two centres of power: one inland, and one on the coast.
277
2
03
Of these six cities, two were very important: Puhar or Kaveripattinam, the port of the Cholas, and
5
41
Madurai, the capital of the Pandyas.
th - 9
Q Source: Additional Research: Page 139: TN 11 Standardm Textbook

i l .co
34 Which of these developments can be attributed tog ma Vedic age, as against the practice in the
Later
Vedic age?
a 7@
is r
1. Women lost their political rights of attending assemblies.
2. Child marriage were banned andithe m
t practice of sati was absent.
3. Women were not allowed to u l k
- p wear garments made of cotton.
sra using the codes below.
Select the correctianswer
A. 1 onlyit M
B. 2u lk 3 only
and
P
C. 3 only
D. 2 only

Your Answer : D
Correct Answer : A

Answer Justification :

Background: The Rig Vedic society was patriarchal. But, the condition of women was much better
in this period that in the Later vedic age.

Women were given equal opportunities as men for their spiritual and intellectual
development. There were women poets like Apala, Viswavara, Ghosa and Lopamudra during
the Rig Vedic period.

www.insightsias.com 25
© Insights Active Learning | All rights reserved - 78168. You may not reproduce, distribute or exploit the contents in any form without
written permission by copyright owner. Copyright infringers may face civil and criminal liability
Total Marks : 200
TEST - 3(MOCK TEST - 3) Mark Scored : 89.33

Women could even attend the popular assemblies. There was no child marriage and the
practice of sati was absent.

Justification: Statement 1: In the family, the power of the father increased during the Later Vedic
period.

There was no improvement in the status of women. They were still considered inferior and
subordinate to men.

Women also lost their political rights of attending assemblies.

Statement 2: Child marriages had become common during the later vedic period.

7 of cotton and
Statement 3: In both periods, men and women wore upper and lower garments made
7
wool. A variety of ornaments were used by both men and women. 2 2
5 03
th
Q Source: Revision Previous tests: Page 29: TN 11 Standard Textbook
9 41
-
m
i l .co
35 With reference to Ancient texts, consider the following statements.
1. The Brahmanas are the law texts under Manusmriti m a
that helped establish the predominance of
Brahmana community. 7 @g
sra and sacrificial ceremony.
2. Upanishads are treatises relating to prayer
i
3. Aranyakas are called forest bookstand
i m they deal with mysticism.
4. Yajurveda deals with music, u l k
chanting and medicine.
- p
sr a
Select the correct answer using the codes below.
M i
A. 1 and 2tonly
k i
ul
B. 3 only
C. P1, 3 and 4 only
D. 2 and 4 only

Your Answer : A
Correct Answer : B

Answer Justification :

Justification: Besides the Vedas, there are other sacred works like the Brahmana, the Upanishads,
the Aranyakas and the epics Ramayana and Mahabharata.

The Brahmanas are the treatises relating to prayer and sacrificial ceremony.

The Upanishads are philosophical texts dealing with topic like the soul, the absolute, the origin of
the world and the mysteries of nature.

The Aranyakas are called forest books and they deal with mysticism, rites, rituals and sacrifices.

www.insightsias.com 26
© Insights Active Learning | All rights reserved - 78168. You may not reproduce, distribute or exploit the contents in any form without
written permission by copyright owner. Copyright infringers may face civil and criminal liability
Total Marks : 200
TEST - 3(MOCK TEST - 3) Mark Scored : 89.33

Yajurveda is a compilation of ritual offering formulas that were said by a priest while an individual
performed ritual actions such as those before the yajna fire.

Q Source: Revision Previous tests: Page 28: TN 11th Standard Textbook

36 Irula people
1. Are a tribe of the Andaman and Nicobar Islands
2. Practice iron smelting as the main occupation
3. Do not speak any known dialect

Select the correct answer using the codes below.


A. 1 and 2 only
B. 3 only
C. 2 only
D. None of the above
7
Your Answer : D 2 27
0 3
Correct Answer : D 5
41
Answer Justification : -9
. com
Justification: Statement 1: Irula is an ethnic group a l
ofiIndia. They inhabit the area of the Nilgiri
m
@g
mountains, in the states of Tamil Nadu and Kerala.
7
isra
Statement 2: Traditionally, the main occupation of the Irulas has been snake and rat catching. They
also work as labourers (coolies) in m
itthe fields of the landlords during the sowing and harvesting
u l k
seasons or in the rice mills. Fishing is also a major occupation.
- p
Statement 3: People sr a
of Irula ethnicity are called Irular, and speak Irula, which belongs to the
i
k itM
Dravidian family.

ul Negrito tribes in the Andaman Islands who have retained their language, Irular speak
Pthe
Unlike
the Irula language, a Dravidian language that is closely related to Tamil, Yerukala, Sholaga and
other Tamil languages.

Q Source: Additional Research: Page 90: TN 11th Standard Textbook

37 Buddhamitra was known for


1. Writing virasoliyam which is a work on Tamil Grammar
2. Erecting inscriptions on images of bodhisattvas and the Buddha near Ganges river

Which of the above is/are correct?


A. 1 only
B. 2 only
C. Both 1 and 2
D. None

www.insightsias.com 27
© Insights Active Learning | All rights reserved - 78168. You may not reproduce, distribute or exploit the contents in any form without
written permission by copyright owner. Copyright infringers may face civil and criminal liability
Total Marks : 200
TEST - 3(MOCK TEST - 3) Mark Scored : 89.33

Your Answer :
Correct Answer : C

Answer Justification :

Justification: Buddhamitra was a Buddhist nun from India during the Kushan Empire.

Statement 1: She is remembered because of dated inscriptions on images of bodhisattvas and the
Buddha that she erected in three cities near the Ganges river.

They mark her success in attracting money and patronage to the Sarvastivada, the sect of Buddhism
to which she belonged.

Statement 2: Virasoliyam attempts to find synthesis between Sanskrit and Tamil grammar.

There were other books written on Tamil grammar as well like Yapperungalam and
Yapperungalakkarigai by the Jain ascetic Amirtasagara.
7
2 27
th
Q Source: Additional Research: Page 150: TN 11 Standard Textbook
0 3
5
41
38 Adipurana written by Gunabhadra is a work on the -9
. com
ai l
A. Commentary of esoteric aspects found in the Vedas
gm
7@
B. Life stories of various Jain saints
C. Sanskrit Grammar
s r a
D. mi
Verses used in sacrificial ceremony
i t
Your Answer : p ulk
a -
Correct Answer : B
i sr
M
kit
Answer Justification
l :
Pu
Justification: It is a 9th century Sanskrit poem composed by Jinasena, a Digambara monk.

It deals with the life of Rishabhanatha, the first Tirthankara, and other saints.

Learning: Amogavarsha I, who was a Jain patronized many Jain scholars.

His teacher Jinasena composed Parsvabhudaya, a biography of Parsva in verses.

Sakatayana wrote the grammer work called Amogavritti. And, the great mathematician of this
period, Viracharya was the author of Ganitasaram.

Q Source: Page 137: TN 11th Standard Textbook

39 The images of Shiva represented in various forms like Nataraja, Gangadhara, Ardhanareesvara,
Somaskanda and Trimurthi can be found in

www.insightsias.com 28
© Insights Active Learning | All rights reserved - 78168. You may not reproduce, distribute or exploit the contents in any form without
written permission by copyright owner. Copyright infringers may face civil and criminal liability
Total Marks : 200
TEST - 3(MOCK TEST - 3) Mark Scored : 89.33

A. Elephanta caves
B. Ajanta caves
C. Karla caves
D. Kanheri rock-cut caves

Your Answer :
Correct Answer : A

Answer Justification :

Learning: Elephanta is an island near Bombay. It was originally called Sripuri.

The Portuguese after seeing the large figure of an elephant named it Elephanta.

The sculptural art of the Rashtrakutas reached its zenith in this place.

There is a close similarity between the sculptures at Ellora and those in Elephanta.7They might have
been carved by the same craftsmen. 27 2
3
is1
The most imposing figure of this temple is Trimurthi. The sculpture 4 50
six metre high. It is said to
- 9
represent the three aspects of Shiva as Creator, Preserver and Destroyer.

. com
Q Source: Page 138: TN 11th Standard Textbook
ai l
gm
@
r a7 and Sangam age, what is/are the chief similarities in
40 If you compare the socio-economic life of Cholas
s
i of women in society?
these periods with respect to the position
m
t
lki
1. The practice of Sati was present.
u
p
2. Devadasi system was abolished.
-
i sra
3. Women were not allowed to write or study literature.
4. Love marriage Mwas forbidden.

l k it
u correct answer using the codes below.
SelectPthe
A. 1 only
B. 2, 3 and 4 only
C. 3 and 4 only
D. 1, 2 and 4 only

Your Answer : A
Correct Answer : A

Answer Justification :

Justification: Statement 1 and 4: Love marriage was a common practice in the Sangam age.
Women were allowed to choose their life partners.

However, the life of widows was miserable. The practice of Sati was prevalent in the higher
strata of society.

www.insightsias.com 29
© Insights Active Learning | All rights reserved - 78168. You may not reproduce, distribute or exploit the contents in any form without
written permission by copyright owner. Copyright infringers may face civil and criminal liability
Total Marks : 200
TEST - 3(MOCK TEST - 3) Mark Scored : 89.33

Even in Cholas, practice of ‘sati’ was prevalent among the royal families.

Statement 2: The devadasi system or dancing girls attached to temples emerged during the Chola
period. There is evidence that they were often subjected to harassment by priets and officials.

Statement 3: Women poets like Avvaiyar, Nachchellaiyar, and Kakkaipadiniyar flourished in the
sangam period and contributed to Tamil literature.

Q Source: Page 91 and 149: TN 11th Standard Textbook

41 With reference to Chola Education and Literature, consider the following:


1. Temples and mathas served as the only educational centres.
2. Not only Vedas and Epics, but subjects like mathematics and medicine were also taught.
3. Endowment of lands was made to run educational centres.

7
Select the correct answer using the codes below.
2 27
A. 1 and 2 only
0 3
5
B. 2 and 3 only
41
C. 3 only -9
D. 1, 2 and 3
. com
ai l
Your Answer : B
gm
Correct Answer : B 7 @
isra
Answer Justification :
i t m
ulk
p During this period, besides the temples and mathas as educational
Justification: Statement- 1:

i sra
centres, several educational institutions also flourished. The inscription at Ennayiram,
ThirumukkudalM and Thirubhuvanai provide details of the colleges existed in these places.
l k it
Pu 2 and 3: The development of Tamil literature reached its peak during the Chola period.
Statement
And a lot of it came from the contribution of education and literature to the enrichment of this
culture.

Mathematics and medicine studies showed that Kings encouraged logical and intellectual pursuits
and were not centred entirely on religious studies.

Q Source: Page 149: TN 11th Standard Textbook

42 Vanavaramban, Vanavan, Kuttuvan, Irumporai and Villavar were famous titles assumed by

A. Chera kings
B. Sangam poets
C. Thiruvalluvar, a tamil philosopher and poet
D. Temple priests in Southern Indian dynasties

www.insightsias.com 30
© Insights Active Learning | All rights reserved - 78168. You may not reproduce, distribute or exploit the contents in any form without
written permission by copyright owner. Copyright infringers may face civil and criminal liability
Total Marks : 200
TEST - 3(MOCK TEST - 3) Mark Scored : 89.33

Your Answer : C
Correct Answer : A

Answer Justification :

Learning: Hereditary monarchy was the form of government during the Sangam period.

The Chera kings assumed titles like Vanavaramban, Vanavan, Kuttuvan, Irumporai and Villavar, the
Chola kings like Senni, Valavan and Killi and the Pandya kings Thennavar and Minavar.

Each of the Sangam dynasties had a royal emblem – carp for the Pandyas, tiger for the Cholas and
bow for the Cheras.

Q Source: Page 89: TN 11th Standard Textbook

43 Consider the following statements with reference to the South Indian dynasties.
7
1. Arikkamedu was a seaport during the Sangam period.
2 27
2. Uraiyur was famous for pearls.
5 03
3. Lord Murugan was the primary deity of the Sangam age. 1 4
-9
Select the correct answer using the codes below.
. com
A. 1 only
ai l
B. 2 and 3 only m
C. 3 only 7 @g
a
D. 1, 2 and 3 isr m
l k it
Your Answer : A
p u
Correct Answer : D
a -
i sr
t M
Answer Justification :
u lki
P
Justification: Statement 1: External trade was carried between South India and the Greek
kingdoms.

The port city of Puhar became an emporium of foreign trade, as big ships entered this port
with precious goods.

Other ports of commercial activity include Tondi, Musiri, Korkai, Arikkamedu and
Marakkanam.

Statement 2: Uraiyur was an ancient Chola city with a fortress and city wall on the southern banks
of the river Kaveri. It was known for pearls.

Statement 3: The worship of Murugan was having an ancient origin and the festivals relating to God
Murugan was mentioned in the Sangam literature. He was honoured with six abodes known as
Arupadai Veedu.

www.insightsias.com 31
© Insights Active Learning | All rights reserved - 78168. You may not reproduce, distribute or exploit the contents in any form without
written permission by copyright owner. Copyright infringers may face civil and criminal liability
Total Marks : 200
TEST - 3(MOCK TEST - 3) Mark Scored : 89.33

Q Source: Page 92: TN 11th Standard Textbook

44 Consider the following about the visit of the famous Chinese traveller, Fahien.
1. He visited India during the reign of Chandragupta II.
2. He stayed for over twenty years in India.
3. He came to India by sea route.
4. His main purpose of visiting India was to study its political and social system.
5. He studied Sanskrit in Patliputra.

Select the correct answer using the codes below.


A. 2, 3 and 4 only
B. 1, 2 and 5 only
C. 1 and 5 only
D. 1, 2, 4 and 5 only

7
27
Your Answer : D
Correct Answer : C 3 2
5 0
41
Answer Justification :
-9
m
i l .co in India, he spent six years in the Gupta
Justification: Statement 1 and 2: Out of his nine years stay
empire. a m
@ g
7 through Khotan, Kashgar, Gandhara and Punjab.
Statement 3: He came to India by the landaroute
isr
it m
ulk Kanauj, Sravasti, Kapilavastu, Kusinagara, Pataliputra, Kasi
He visited Peshawar, Mathura,
p
- other places.
and Bodh Gaya among
i sra
M
kit by the sea route, visiting on the way Ceylon and Java.
He returned
l
Pu
Statement 4: The main purpose of his visit was to see the land of the Buddha and to collect Buddhist
manuscripts from India.

Statement 5: He stayed in Pataliputra for three years studying Sanskrit and copying Buddhist texts.

Q Source: Page 102: TN 11th Standard Textbook

45 Hiuen Tsang portray the social life in the times of Harsha. Consider the following with reference to it.
1. He mentions that the Sudras were forbidden from agriculture.
2. As per his documentation, the dead could not be subjected to water burial.
3. Dowry system was practiced in the empire.

Select the correct answer using the codes below.


A. 1 only
B. 2 and 3 only

www.insightsias.com 32
© Insights Active Learning | All rights reserved - 78168. You may not reproduce, distribute or exploit the contents in any form without
written permission by copyright owner. Copyright infringers may face civil and criminal liability
Total Marks : 200
TEST - 3(MOCK TEST - 3) Mark Scored : 89.33

C. 3 only
D. 1 and 2 only

Your Answer : C
Correct Answer : C

Answer Justification :

Justification: The fourfold division of the society – Brahmin, Kshatriya, Vysya and Sudra – was
prevalent.

Statement 1: The Brahmins were the privileged section of the society and they were given land
grants by the kings. Hiuen Tsang mentions that the Sudras practiced agriculture.

Statement 2: Hiuen Tsang mentions three ways of disposal of the dead – cremation, water burial
and exposure in the woods.
7
2 27
Statement 3: The position of women was not satisfactory. The institution of Swyamvara (the choice
of choosing her husband) had declined. 0 3
1 5
- 94
m
.cowas also prevalent.
Remarriage of widows was not permitted, particularly among the higher castes. The system of
dowry had also become common. The practice ofilsati
m a
@g
Learning: Trade and commerce had declined
s r a7 during Harsha’s period, which is is evident from the
mof
decline of trade centres, less number i coins, and slow activities of merchant guilds.
it
p ulk
a - in turn affected the handicrafts industry and agriculture.
The decline of trade
sr
M i
it
lk there was no large-scale demand for goods, the farmers began to produce only in a
Since
u
Plimited way. This led to the rise of self-sufficient village economy.

Q Source: Page 117: TN 11th Standard Textbook

46 IFAW (International Fund for Animal Welfare)


1. is a non-profit organization
2. provides hands-on assistance to animals
3. campaigns to end commercial whaling
4. was founded by the World Wildlife Fund (WWF)

Select the correct answer using the codes below.


A. 1 and 4 only
B. 2 and 3 only
C. 1, 2 and 3 only
D. 2 and 4 only

www.insightsias.com 33
© Insights Active Learning | All rights reserved - 78168. You may not reproduce, distribute or exploit the contents in any form without
written permission by copyright owner. Copyright infringers may face civil and criminal liability
Total Marks : 200
TEST - 3(MOCK TEST - 3) Mark Scored : 89.33

Your Answer :
Correct Answer : C

Answer Justification :

Justification: The International Fund for Animal Welfare (IFAW) is one of the largest animal
welfare and conservation charities in the world.

The group's declared mission is to "rescue and protect animals around the world.

The International Fund for Animal Welfare (IFAW) was founded by a small group of people in
1969, to stop the commercial hunt for seal pups in Canada.

The International Tiger Day (also known as Global Tiger Day) is celebrated every year on 29
July to raise awareness for tiger conservation.
7
2 27
3
50 day.
The IFAW recently organized major events on International Tiger conservation
1
4
You can read more about it here http://www.ifaw.org/united-states/about-ifaw -9
. com
Q Source: As mentioned above ai l
gm
7 @
47 Consider the following:
isra
k i tm
1. They developed Vesara style of architecture.
lat Aihole, Badami and Pattadakal.
pu centralized.
2. Their structural temples exist
3. Their administration was- highly
i sra
M
The above most accurately refer to?
l k
A. Cholas
it
B. P
u
Chalukyas
C. Cheras
D. Pandyas

Your Answer : B
Correct Answer : B

Answer Justification :

Justification: Administration: The Chalukya administration was highly centralized unlike that of
the Pallavas and the Cholas. Village autonomy was absent under the Chalukyas.

Architecture: The Chalukyas were great patrons of art. They developed the vesara style in the
building of structural temples. However, the vesara style reached its culmination only under the
Rashtrakutas and the Hoysalas.

www.insightsias.com 34
© Insights Active Learning | All rights reserved - 78168. You may not reproduce, distribute or exploit the contents in any form without
written permission by copyright owner. Copyright infringers may face civil and criminal liability
Total Marks : 200
TEST - 3(MOCK TEST - 3) Mark Scored : 89.33

Cave temple architecture was also famous under the Chalukyas. Their cave temples are found
in Ajanta, Ellora and Nasik.

The best specimens of Chalukya paintings can be seen in the Badami.

Q Source: Additional Research: Page 134: 11th TN Textbook

48 With reference to Ancient India, mahajanapada rulers imposed taxes on

A. Crops and crafts persons only


B. Goods, crops, herders, hunter gatherers and crafts persons
C. Goods and royal services only
D. Hunter gatherers and crafts persons only

7
Your Answer : B
2 27
Correct Answer : B 0 3
5
41
Answer Justification : -9
om
a il.c
Background: As they were building huge forts and maintaining big armies, they needed more
resources. m
7 @g
s a
Instead of depending on occasional giftsrbrought by people, as in the case of the raja of the
i
m taxes.
janapadas, they started collecting regular
i t
Learning: Taxes on crops p ulkthe most important. This was because most people were farmers.
were
a -
isr
M
t taxes on crafts persons as well. These could have been in the form of labour. For
There iwere
l k
Pu
example, a weaver or a smith may have had to work for a day every month for the king.

Herders were also expected to pay taxes in the form of animals and animal produce.

There were also taxes on goods that were bought and sold, through trade.

And hunters and gatherers also had to provide forest produce to the raja.

Q Source: Revision previous tests: Page 59-60: 6th NCERT

49 Dakshinapatha and Uttarapatha were

A. Two great highways that have connected different parts of the sub-continent since the Iron
Age

www.insightsias.com 35
© Insights Active Learning | All rights reserved - 78168. You may not reproduce, distribute or exploit the contents in any form without
written permission by copyright owner. Copyright infringers may face civil and criminal liability
Total Marks : 200
TEST - 3(MOCK TEST - 3) Mark Scored : 89.33

B. Pali transcriptions of two different sects of Buddhism


C. Extensions of Chola Empire in Sri Lanka and Northern India respectively
D. The largest gopurams of Brihadeshwara temple made under Rajaraja I

Your Answer : A
Correct Answer : A

Answer Justification :

Learning: Dakshinapatha is a historical which has been used to describe the great southern
highway in India, traveling from Magadha to Pratishthana.

Uttarapatha or the great northern road ran from Taxila in Afganisthan, through the modern
Punjab up to the western coast of Yamuna.

7 it passed on to
Following the course of Yamuna, it went southwards up to Mathura, from2there
7
2
Ujjain in Malwa and to Broach on western coast. 03 5
1
- 94
m Shri Satakarni and other
The most important ruler of the Satavahanas, Gautamiputra
o
l. c
Satavahana rulers were known as lords of the dakshinapatha.
i
m a
Q Source: Additional Research: Page 99: a
11
@g
7th TN Textbook
isr
it m
p ulk
50 Consider the following about Traditional Knowledge Digital Library (TKDL), an Indian initiative.

a -
1. It is to protect Indian traditional knowledge from exploitation by wrongful patents mainly at
International Patent r
is Offices.
2. It is maintained M
t a digitized format and is available only in old Sanskrit so as to avoid the misuse
in
k i
ul discoveries.
of ancient
P
Which of the above is/are correct?
A. 1 only
B. 2 only
C. Both 1 and 2
D. None

Your Answer : A
Correct Answer : A

Answer Justification :

Justification: It is an organisation of the Council of Scientific and Industrial Research.

TKDL contains Indian traditional medicine knowledge available in public domain and pertains
to classical/ traditional books related to Ayurveda, Unani and Siddha in a digitized format and

www.insightsias.com 36
© Insights Active Learning | All rights reserved - 78168. You may not reproduce, distribute or exploit the contents in any form without
written permission by copyright owner. Copyright infringers may face civil and criminal liability
Total Marks : 200
TEST - 3(MOCK TEST - 3) Mark Scored : 89.33

is available in five international languages (English, French, German, Spanish and Japanese).

India’s rich and time-tested traditional medicinal knowledge which exists in languages such as
Sanskrit, Hindi, Arabic, Persian, Urdu, Tamil etc. is neither accessible nor comprehensible for
patent examiners at the international patent offices.

So, let’s say if some corporation patents the use of termuric in curing certain chronic diseases, it’ll
amount to piracy of our traditional knowledge.

If this information about ancient knowledge is already made available to patent offices for pre-
verification in different languages beforehand, such misuse can be stopped.

Q Source:
http://www.thehindu.com/todays-paper/ancient-knowledge-hub-faces-grey-future/article17625761.e
ce
7
2 27
3
51 Consider the following about the Senior Citizens Welfare Fund.
1 50
94
1. It was established along with the Central Social Welfare Board (CSWB).
-
2. It will be administered by an Inter-Ministerial Committee. m
.co character as a welfare fund.
3. The fund shall not be audited by the CAG due to its fundamental
il
m a
g
7@
Select the correct answer using the codes below.
A. 2 only r a
B. 1 and 3 only m is
C. 1 and 2 only l k it
D. 2 and 3 only
- pu
i sra
t M: A
Your Answer : C
k i
Correct Answer
l
Pu
Answer Justification :

Justification: Statement 1: CSWF was established in the 1950s, so 1 is wrong. You can read about
CSWB here http://www.cswb.gov.in/index1.asp?linkid=230&langid=1

The Centre brought in Senior Citizens’ Welfare Fund Act, 2015 (SCWF) as part of the Finance Act,
2015, which mandates transfer of unclaimed amounts of policyholders to the fund (SCWF) after a
period of 10 years.

Statement 2: The fund will be administered by an Inter-Ministerial Committee, headed by a


Chairperson. The Committee will be competent to spend money from the fund for satisfying various
objectives.

Statement 3: The accounts of the fund will be open to audit by CAG, regularly. The Central
Government will present the annual report and the one furnished by CAG to be laid before the
Parliament.

www.insightsias.com 37
© Insights Active Learning | All rights reserved - 78168. You may not reproduce, distribute or exploit the contents in any form without
written permission by copyright owner. Copyright infringers may face civil and criminal liability
Total Marks : 200
TEST - 3(MOCK TEST - 3) Mark Scored : 89.33

Q Source: http://www.insightsonindia.com/2017/07/26/insights-daily-current-affairs-26-july-2017/

52 National Anti-profiteering Authority will look after

A. Enforcing anti-trust or competitive laws in the industry on the lines of Competition


Commission of India (CCI)
B. Consumer grievances arising due to complaints raised by unreasonable pricing of goods and
services
C. Ensuring that benefits of a reduction in tax on supply of goods or services flow to the
consumers
D. None of the above

Your Answer : C
Correct Answer : C

7
27
Answer Justification :
3 2
50credit.
Concept: In the GST mechanism, the producers are provided an input tax
1
4
-9
It works this way. Suppose, a producer requires INR o mworth of cotton to produce a shirt,
80
l . c
which is finally processed and priced at INR 100iin the market.
m a
7 @g
sra to pay taxes at two places – a) while purchasing the
In this process, the producer will have
i
m the final produce – shirt.
raw material, and b) while selling
i t
ulk
-p
i sra (tax on tax) is avoided by an input tax credit mechanism. Under this, the
This double taxation
producers are compensated for any additional taxes they would have paid.
M
l k it
P u
Justification: When constituted by the GST Council, the National Anti-profiteering Authority shall
be responsible for applying anti-profiteering measures in the event of a reduction in rate of GST on
supply of goods or services.

It will also check if the benefit of input tax credit is not passed on to the recipients by way of
commensurate reduction in prices.

Learning: In the event the National Anti-profiteering Authority confirms the necessity of applying
anti-profiteering measures, it has the power to order the business concerned to reduce its prices or
return the undue benefit availed along with interest to the recipient of the goods or services.

If the undue benefit cannot be passed on to the recipient, it can be ordered to be deposited in
the Consumer Welfare Fund.

In extreme cases the National Anti-profiteering Authority can impose a penalty on the

www.insightsias.com 38
© Insights Active Learning | All rights reserved - 78168. You may not reproduce, distribute or exploit the contents in any form without
written permission by copyright owner. Copyright infringers may face civil and criminal liability
Total Marks : 200
TEST - 3(MOCK TEST - 3) Mark Scored : 89.33

defaulting business entity and even order the cancellation of its registration under GST.

Q Source: http://pib.nic.in/newsite/PrintRelease.aspx?relid=169000

53 Consider the following about electoral bonds that were introduced in the 216-17 annual budget.
1. They will be an interest-paying debt instrument.
2. They cannot be sold by banks.
3. They can be deposited in notified accounts of political parties.
4. The bonds will not carry the name of the donor.

Select the correct answer using the codes below.


A. 1 and 2 only
B. 3 and 4 only
C. 1, 2 and 3 only
D. 2 and 3 only 7
2 27
Your Answer : B 0 3
5
Correct Answer : B 41
-9
Answer Justification :
. com
ai l
gm to political parties.
Concept: Electoral bonds are to be used for donations
7@
sra over donations through cheques is that many donors
The advantage of the electoral bond system
i
expressed reluctance to use cheques
k i tmbecause it becomes transparent and leads to political vendetta
by rivals. ul p
a -
sr electoral bond mechanism announced in the Budget, the proposed bonds will
Features: As per the
i
M
resemble a promissory note and not an interest-paying debt instrument.
l k it
Pu
They will be sold by authorised banks and can be deposited in notified accounts of political
parties within the duration of their validity.

The bonds will not carry the name of the donor and routing of the money through banks will
ensure that only tax paid money comes into the political system.

The government is planning to go ahead with electoral bonds even if consensus eludes on the issue
as political parties have not come up yet with any suggestion on the proposal.

Q Source: http://www.insightsonindia.com/2017/07/25/insights-daily-current-affairs-25-july-2017/

54 Anti-dumping duty is usually applied in order to


1. Contain trade distortion
2. Increase imports of capital goods

www.insightsias.com 39
© Insights Active Learning | All rights reserved - 78168. You may not reproduce, distribute or exploit the contents in any form without
written permission by copyright owner. Copyright infringers may face civil and criminal liability
Total Marks : 200
TEST - 3(MOCK TEST - 3) Mark Scored : 89.33

3. Sterilize foreign capital inflows

Select the correct answer using the codes below.


A. 1 and 2 only
B. 3 only
C. 1 only
D. 2 and 3 only

Your Answer : C
Correct Answer : C

Answer Justification :

Justification: Statement 1: Suppose a nation like China is dumping (exporting) its goods at
exceptionally cheap prices so that it can capture the market. Since these cheap prices can’t be
justified by market forces alone, the Indian government must apply a duty (like a tax) on the
product to raise its price. This is to shield the domestic producers. 77
3 22
0
Statement 2: Capital goods are usually not dumped by nations, since the5industry is highly
4 1
-9
segmented and its hard to dump products at cheap prices.

Statement 3: It has nothing to do with FDI or FII.


. com
ai l
Learning: India has initiated over two hundred g m
anti-dumping investigations between 2012 and
@
a7 and Indonesia. During the period, maximum
2017 against various countries, including China
number of cases were against China. isr

k i tm
l levy was imposed include chemicals, fibre boards, glass &
Product categories on whichuthe
p
- and steel.
glassware, pharmaceuticals
r a
M is
Q Source: http://www.insightsonindia.com/2017/07/25/insights-daily-current-affairs-25-july-2017/
l k it
Pu
55 If China were to takeover Doklam plateau, which region in or near India would be most affected by
this move?

A. Leh and Ladakh


B. Siliguri Corridor
C. India-Uttarakhand Border
D. Sundarban delta

Your Answer : B
Correct Answer : B

Answer Justification :

Learning: China is building a road project in the region which as per Bhutan would run from the
town of Dokola to the Bhutanese army camp at Zompelri.

www.insightsias.com 40
© Insights Active Learning | All rights reserved - 78168. You may not reproduce, distribute or exploit the contents in any form without
written permission by copyright owner. Copyright infringers may face civil and criminal liability
Total Marks : 200
TEST - 3(MOCK TEST - 3) Mark Scored : 89.33

This is seen by Bhutan as a direct violation of the agrements between two nations.

India has said the Chinese road project threatens its access to the Siliguri corridor that connects
our mainland to north-eastern states.

7
2 27
0 3
5
41
-9
The tense standoff between India and China over the Doklam omplateau has only escalated, raising
concerns in both capitals of an all-out military conflict.il.c
m a
@g
Q Source: http://www.insightsonindia.com/2017/07/25/insights-daily-current-affairs-25-july-2017/
7
isra
i mnews is between
tin
56 The Mekedatu dispute recently seen
ul k
p
- Nadu
a
A. Karnataka andrTamil
M
B. Government isof India and Government of Sri Lanka
k it community from Kerala and Northern province of Sri Lanka
C. Fishermen
l
D. P u Pradesh, Telangana and Karnataka
Andhra

Your Answer : A
Correct Answer : A

Answer Justification :

Learning: Karnataka intends to build a reservoir across river Cauvery near Mekedatu in
Kanakapura taluk.

It was first proposed along with Shivanasamudra hydro power project in 2003 with an
intention to use the water for a hydro power station and supply drinking water to Bengaluru
city.

However, Tamil Nadu objected saying Karnataka had not sought prior permission for the
project. Its argument was that the project would affect the flow of Cauvery water to Tamil

www.insightsias.com 41
© Insights Active Learning | All rights reserved - 78168. You may not reproduce, distribute or exploit the contents in any form without
written permission by copyright owner. Copyright infringers may face civil and criminal liability
Total Marks : 200
TEST - 3(MOCK TEST - 3) Mark Scored : 89.33

Nadu.

Q Source: http://www.insightsonindia.com/2017/07/26/insights-daily-current-affairs-26-july-2017/

57 Why the government has introduced the Central Road Fund (Amendment) Bill, 2017 in the Lok
Sabha?

A. To provide for development of national waterways (NWs) from this fund


B. To add state and local highways to the provision list of this fund
C. To upgrade road construction and maintenance in the category of plan expenditure
D. To raise revenue for road development via foreign direct investment

Your Answer : A
Correct Answer : A
7
Answer Justification :
2 27
0 3
Background: The fund is utilised for the development and maintenance
4 15of National highways,
- 9 bridges and other safety
State roads, Rural roads and for provision of road overbridges/under
features at unmanned Railway Crossings. om
a il.c
g m
7of@customs on petrol and high speed diesel oil.
In order to mobilise the fund, the Central Road Fund Act 2000 proposed to levy and collect by
sr a
way of cess, a duty of excise and duty
i
k i tm
Learning: The bill seeks top ul the Central Road Fund Act, 2000, to allocate 2.5% of the
amend
a -
proceeds of CRF for development and maintenance of NWs and a reduction in the share provided
i sr
for development of National Highways.

k itM
l http://www.insightsonindia.com/2017/07/25/insights-daily-current-affairs-25-july-2017/
Pu
Q Source:

58 The Banking Regulation (Amendment) Bill, 2017 seeks to address which of these major issues in the
banking sector?

A. Non-performing assets
B. Lack of competition in public sector banks
C. Increasing participation of women in top tier management of banks
D. Financial inclusion in rural areas

Your Answer : A
Correct Answer : A

Answer Justification :

Background: Stressed assets (NPAs or bad debt) in the banking system have reached unacceptably
high levels and hence, urgent measures were required for their speedy resolution.

www.insightsias.com 42
© Insights Active Learning | All rights reserved - 78168. You may not reproduce, distribute or exploit the contents in any form without
written permission by copyright owner. Copyright infringers may face civil and criminal liability
Total Marks : 200
TEST - 3(MOCK TEST - 3) Mark Scored : 89.33

Therefore, the government considered it necessary to make provisions in the Banking


Regulation Act, 1949 for authorizing the Reserve Bank of India to issue directions to any
banking company or banking companies to effectively use the provisions of the Insolvency and
Bankruptcy Code, 2016 for timely resolution of stressed assets.

Learning: The 2017 bill gives powers to the Reserve Bank of India (RBI) to ask any bank to initiate
insolvency proceedings and give directions for resolution of stressed assets.

The RBI would also be empowered to issue other directions for resolution, appoint or approve for
appointment, authorities or committees to advise the banking companies for stressed asset
resolution.

Q Source: http://www.insightsonindia.com/2017/07/25/insights-daily-current-affairs-25-july-2017/

59 In 2015, under the Indradhanush plan, the government had announced capital infusion of over
7agencies had
2to7
seventy thousand crores in public sector banks for four years. However, credit rating
pointed out that the sum was insufficient because banks needed additional funds2
5 03
1. Meet Basel-III norms 1
2. Tackle the menace of rising bad loans 94 -
. com
Which of the above is/are correct?
a i l
A. 1 only m
B. 2 only
7 @g
C. Both 1 and 2
isra
D. None tm i
lk
Your Answer : B - pu
Correct Answer : Csra
M i
l k it
Answer Justification :
P u
Background: "Basel III" is a comprehensive set of reform measures, developed by the Basel
Committee on Banking Supervision, to strengthen the regulation, supervision and risk management
of the banking sector.

It is an improvement over Base I and Basel II norms released much earlier.

These norms may include reducing bank investment in risky assets, increasing their capital
base, strengthening management etc.

Justification: Statement 1: Rating agencies pointed that apart from PSBs face lack of competition
and if business as usual continues, they will face dearth of funds for expansion because:

Private sector will give them a tough competition in coming years

www.insightsias.com 43
© Insights Active Learning | All rights reserved - 78168. You may not reproduce, distribute or exploit the contents in any form without
written permission by copyright owner. Copyright infringers may face civil and criminal liability
Total Marks : 200
TEST - 3(MOCK TEST - 3) Mark Scored : 89.33

Basel III norms required more funds to absorb greater risks for banks

Statement 2: The background has been explained in another question on banking amendment bill.

Q Source: http://www.insightsonindia.com/2017/07/29/insights-daily-current-affairs-29-july-2017/

60 Methanol is a promising fuel for waterways in India because


1. It does not have any smog based emissions.
2. It is cheaper than most fossil fuels.
3. India manufactures all its requirement of methanol by domestic production.

Select the correct answer using the codes below.


A. 1 and 2 only
B. 2 and 3 only
C. 2 only 7
D. 3 only
2 27
0 3
5
Your Answer : A 41
Correct Answer : C -9
. com
Answer Justification : ai l
gm
Justification: Across the world, methanol is @
7emerging as a clean, sustainable transportation fuel of
s r a
the future.
m i
kit
Methanol can be blended withlgasoline in low-quantities and used in existing road vehicles, or it can
u
- pblends.
be used in high-proportion

i sra
t M
Statement 1: Methanol is a clean-burning fuel that produces fewer smog-causing emissions such as
lk i
sulphur oxides (SOx), nitrogen oxides (NOx) and particulate matter. This can improve air quality
u
P human health issues.
and related

Statement 2 and 3: It is clean, cheaper than fossil fuels and a good substitute for heavy fuels. India
imports methanol from Saudi Arabia and Iran at present. Technology is also being commercialized
to use methanol as a diesel substitute.

Learning: Methanol is most commonly produced on a commercial scale from natural gas. It can
also be produced from renewable sources such as biomass and recycled carbon dioxide.

As a high-octane vehicle fuel, methanol offers excellent acceleration and power. It also
improves vehicle efficiency.

The government has asked the think-tank NITI Aayog to study the automobile standards
developed in China to use methanol as an alternative fuel.

www.insightsias.com 44
© Insights Active Learning | All rights reserved - 78168. You may not reproduce, distribute or exploit the contents in any form without
written permission by copyright owner. Copyright infringers may face civil and criminal liability
Total Marks : 200
TEST - 3(MOCK TEST - 3) Mark Scored : 89.33

Experts believe that Methanol economy will help India use its vast reserves of coal while
driving import substitution.

Q Source; http://www.insightsonindia.com/2017/08/01/insights-daily-current-affairs-01-aug-2017/

61 E-RaKAM is a first-of-its-kind initiative that


1. Leverages technology to connect farmers of the smallest villages to the biggest markets of the
world
2. Establishes grading and standardization centres for agricultural produce in the remotest locations
of the country to promote food processing

Which of the above is/are correct?


A. 1 only
B. 2 only
C. Both 1 and 2 7
D. None 2 27
0 3
5
Your Answer : A 41
Correct Answer : A -9
. com
Answer Justification : ai l
gm
@
7 bringing together the farmers, FPOs, PSUs, civil
Justification: E-RaKAM is a digital initiative
r a
is to ease the selling and buying process of agricultural
supplies and buyers on a single platform
m
products. it
p ulk
a -
Under this initiative, e-RaKAM centres are being developed in a phased manner throughout the
i r
country to facilitatesfarmers for online sale of their produce.

k i ta M
u l joint initiative by state-run-auctioneer MSTC and Central Warehousing Corporation
The portal is
P
arm CRWC.

Q Source: http://www.insightsonindia.com/2017/08/02/insights-daily-current-affairs-02-aug-2017/

62 Who is a Parliamentary secretary?

A. Assists a Minister in his or her duties


B. Heads the Lok Sabha secretariat
C. Discharges the functions of speaker when she is on leave
D. In charge of convening and summoning a session of the Parliament

Your Answer : A
Correct Answer : A

Answer Justification :

www.insightsias.com 45
© Insights Active Learning | All rights reserved - 78168. You may not reproduce, distribute or exploit the contents in any form without
written permission by copyright owner. Copyright infringers may face civil and criminal liability
Total Marks : 200
TEST - 3(MOCK TEST - 3) Mark Scored : 89.33

Learning: A Parliament Secretary is similar to a Minister of State who assists a Minister in his or
her duties. They come next to cabinet ministers and ministers of state in the hierarchy of ministerial
executive.

Recently there was a controversy over the 21 AAP MLAs who were appointed Parliamentary
Secretaries to Ministers of the Delhi government.

It raises the issue whether they should be considered as ministers, and if so, should the cap of 15%
for the size of council of ministers should apply to them.

Q Source: Frequently see in news

63 Consider the following statements about Liquid nitrogen, that was in news for some time.
1. It starts to boil once it reaches room temperature.
2. It is an inflammable gas.
3. It is banned for use as an additive in frozen food in India.
7
2 27
Select the correct answer using the codes below. 0 3
5
A. 1 only
41
B. 2 and 3 only -9
C. 3 only
. com
D. None of the above ai l
gm
Your Answer : B 7 @
Correct Answer : D isra
i t m
Answer Justification :
p ulk
a -
i sr 1: Liquid nitrogen, which has a boiling point of -195.8 degree Celsius, is
Justification: Statement
t M gastronomy chefs to instantly freeze food and drinks.
used by molecular
ulki
P
Recently, it is also being popular in preparing cocktails, as it can immediately chills glasses of
ingredients. It also adds a smoking effect to the drink.

It cools any substance immediately. The rapidity of cooling is also responsible for smaller ice
crystals which gives a smooth texture to the ice cream.

Issues: As it evaporates, liquid nitrogen freezes everything around it, including tissues that come in
contact with it.

According to experts, food and drinks that are prepared with liquid nitrogen should be consumed
only after all the gas has bubbled off.

Statement 2: It is a coloured, odourless, non-inflammable gas. It is a cryogen, which means it is


used to create extremely low temperatures.

www.insightsias.com 46
© Insights Active Learning | All rights reserved - 78168. You may not reproduce, distribute or exploit the contents in any form without
written permission by copyright owner. Copyright infringers may face civil and criminal liability
Total Marks : 200
TEST - 3(MOCK TEST - 3) Mark Scored : 89.33

The gas is so cold that it is capable of rapid freezing any living tissue coming in contact with it.

Statement 3: Liquid nitrogen is permitted as an additive in frozen food as per the guidelines of the
national regulatory body Food Safety and Standards Authority of India (FSSAI).

However, when it comes to the use of liquid nitrogen it is a grey area. There is no clear-cut
guideline for it and generally it is considered to be a novel technique, which can be used by food
business operators.

Q Source: http://www.insightsonindia.com/2017/08/01/insights-daily-current-affairs-01-aug-2017/

64 Under WTO rules, a member country cannot discriminate between its trade partners. Hence
1. A country cannot enter into a free trade agreement with another country without the authorization
of WTO
2. If a special status is granted to a trade partner, it should usually be extended to all members of the
WTO.
7
2 27
Which of the above is/are correct? 0 3
5
A. 1 only
41
B. 2 only -9
C. Both 1 and 2
. com
D. None
ai l
gm
Your Answer : B 7 @
Correct Answer : B isra
i t m
Answer Justification :
p ulk
a -
sr 1: Countries don’t require the permission of WTO for their trade related
Justification: Statement
i
k it M they can move WTO if a country violates the rules of trade set by WTO. They
activities. However,
l
are free to enter into FTAs with other countries.
Pu
Statement 2: This is called as MFN status.

As per a WTO rule, every member of WTO requires to accord this status to other member
countries.

India has already granted this status to all WTO members including Pakistan. Pakistan is yet
to reciprocate.

Most Favoured Nation is a treatment accorded to a trade partner to ensure non-


discriminatory trade between two countries vis-a-vis other trade partners.

MFN, however, allows for countries to enter into regional trade agreements. It also relaxes such
requirements in case of LDCs and other developing countries.

www.insightsias.com 47
© Insights Active Learning | All rights reserved - 78168. You may not reproduce, distribute or exploit the contents in any form without
written permission by copyright owner. Copyright infringers may face civil and criminal liability
Total Marks : 200
TEST - 3(MOCK TEST - 3) Mark Scored : 89.33

Q Source: http://www.insightsonindia.com/2017/08/01/insights-daily-current-affairs-01-aug-2017/

65 The objective of the GST composition scheme is to


1. Reduce duties and taxes on all raw materials to zero
2. Maximize the tax on value added to the final product
3. Reduce the tax compliance cost for small businesses
4. Charge differential taxes for the same product based on different compositions

Select the correct answer using the codes below.


A. 3 only
B. 1 and 2 only
C. 4 only
D. 1, 2 and 3 only

Your Answer : A
7
27
Correct Answer : A
3 2
Answer Justification : 5 0
9 41
Justification: The composition scheme is an alternative method- of levy of tax designed for small
.
taxpayers whose turnover is up to Rs 75 lakh — Rs 50 lakh coinmthe case of eight north-eastern states
and the hilly state of Himachal Pradesh. ai l
gm
@
a7tax burden, the scheme provides that:
To promote small business, and reduce their
sr
m i
it
While a regular taxpayer
p ulkhas to pay taxes on a monthly basis, a composition supplier is
- one return and pay taxes on a quarterly basis.
required to file only
i sra
M
Also,kaitcomposition taxpayer is not required to keep detailed records that a normal taxpayer is
ul
Psupposed to maintain.

Learning: As per the Central GST Act, businesses are eligible to opt for the composition scheme if
a person is not engaged in any inter-state outward supplies of goods and not into making any supply
of goods through an electronic commerce operator who is required to collect tax at source.

Five lakh businesses have opted for the GST composition scheme, which allows them to pay taxes at
a concessional rate and makes compliance easy.

Q Source: http://www.insightsonindia.com/2017/08/01/insights-daily-current-affairs-01-aug-2017/

66 A new representative body — US- India Strategic Partnership Forum (USISPF) — is being set up to
further enhance

A. Business relations between the two countries


B. Defense related strategic relations

www.insightsias.com 48
© Insights Active Learning | All rights reserved - 78168. You may not reproduce, distribute or exploit the contents in any form without
written permission by copyright owner. Copyright infringers may face civil and criminal liability
Total Marks : 200
TEST - 3(MOCK TEST - 3) Mark Scored : 89.33

C. Education and technology related exchange


D. All of the above

Your Answer : B
Correct Answer : A

Answer Justification :

Learning: The non-profit corporation aims to promote bilateral trade and work on creating
meaningful opportunities that have the power to change the lives of citizens.

The body will work closely together with businesses and government leaders to achieve its goals of
driving economic growth, job creation, innovation, inclusion and entrepreneurship.

Q Source: http://www.insightsonindia.com/2017/08/02/insights-daily-current-affairs-02-aug-2017/

7 7
2 2development?
67 The Ministry of Tribal Affairs is implementing the “Scheme of Development of Particularly Vulnerable
Tribal Groups(PVTGs)”. The scheme focuses on which of these key parameters
5 03 of
1. Providing regular unemployment allowances
9 41
2. Economic development of PVTGs through horticulture -
3. Land distribution and land development o m
4. Promotion of traditional and tribal games and sports il.c
m a
7 @g
Select the correct answer using the codes below.
A. 2, 3 and 4 only
isra
B. 1 and 3 only
k i tm
C. 2 and 4 only
p ul
D. 1, 2, 3 and 4
a -
s r
Mi
Your Answer t: C
lki : A
Correct Answer
u
P
Answer Justification :

Justification: It aims at planning their socio-economic development in a comprehensive manner


while retaining the culture and heritage of the community.

Activities under the Scheme includes Livelihood, Employment opportunities and economic
development of PVTGs through Agriculture, Horticulture, Animal Husbandry, Dairy, and
Skilling/ Vocational Training etc.

It further includes Education, Health (Gap filling for effective health service delivery beyond
NHM etc) and Provision of safe drinking water.

Land distribution, land development, Social security, Connectivity (Road and


Telecommunication), Supply of Electricity and irrigation are also taken care of including

www.insightsias.com 49
© Insights Active Learning | All rights reserved - 78168. You may not reproduce, distribute or exploit the contents in any form without
written permission by copyright owner. Copyright infringers may face civil and criminal liability
Total Marks : 200
TEST - 3(MOCK TEST - 3) Mark Scored : 89.33

sports.

Housing and Habitat covers funds for housing for PVTGs, primarily through special assistance
under IAY and additionally through this scheme, for gap filling and for enhancing scope of coverage.

Q Source: Samir/JD/jk (Release ID :168896)

68 A microbial consortium called SONA was recently seen in news. It is related to

A. Bio-remediation of oil spills


B. Identifying gold mines using microbes without physical exploration
C. Improving jute cultivation yield
D. Treatment of sewage sites

Your Answer : A 7
Correct Answer : C
2 27
0 3
5
Answer Justification : 41
-9
Learning: Improved Cultivation and Advanced Retting Exercise
. com for Jute (Jute – ICARE) was
l
launched in 2015 to popularize/introduce some of the ibetter agronomic practices and recently
ma
developed microbial-assisted retting among farmers.
g
@
sr a7
Central Research Institute form i
Research in Jute and Allied Fibres (CRIJAF) has also recently
i t
ulk
developed a microbial consortium called SONA, to enhance the quantity of fibre yield by 20%
as well as its quality p
a - in terms of grade by at least 1 ½ grades.

M isr
l k it the project, regular SMSes are sent in regional languages on improved practices
Also, under
inujute cultivation, to registered farmers.
P
Q Source: PIB: No release ID: Ministry of Textiles: 25 July, 2017

69 The Union Cabinet has given its ex-post facto approval for amendment of the Constitution
(Application to Jammu & Kashmir) Order, 1954 by way of the Constitution (Application to Jammu &
Kashmir) Amendment Order, 2017. This was done in order to

A. Apply GST regime in the State of Jammu & Kashmir


B. Allow alternative communication models like prepaid sim cards in J&K that were earlier
blocked due to security concerns
C. Provide a separate flag to the state of J&K
D. Assign a permanent advisor from the Union government to the state of J&K

Your Answer : A
Correct Answer : A

www.insightsias.com 50
© Insights Active Learning | All rights reserved - 78168. You may not reproduce, distribute or exploit the contents in any form without
written permission by copyright owner. Copyright infringers may face civil and criminal liability
Total Marks : 200
TEST - 3(MOCK TEST - 3) Mark Scored : 89.33

Answer Justification :

Learning: An ordinary amendment to the constitution does not apply necessarily to the state of J&K
which has a separate constitution of its own.

The President needs to give approval to certain amendments about which you will read later in
Laxmikanth.

The approval paves the way for applicability of Goods and Services Tax regime in the State of
Jammu & Kashmir.

Q Source: AKT/VBA/SH (Release ID :169110)

70 The Indian Council for Cultural Relations (ICCR has several Indian Cultural Centres abroad. Consider
the following about them.
1. These cultural centres showcase India’s soft power.
2. There is no Indian Cultural Centre in Central Asia and the Middle east.
277
2
3 community and Indian
3. Activities in these centres are organized with the help of involvement of local
5 0
Diaspora. 1 4
-9
Select the correct answer using the codes below.
. com
A. 1 and 3 only
ai l
B. 2 only m
C. 2 and 3 only
7 @g
D. 1 only sra i
m
Your Answer : A
u lkit
Correct Answer : A -p
i sra
t M
Answer Justification :
lk i
P u
Justification: Statement 1: Soft power of India includes its culture, diaspora, ancient heritage like
yoga and the skills of its population.

Statement 2: During the recent visit of Prime Minister of India to Israel, announcement for the
opening of Indian Cultural Centre in Israel was made.

There are centres in Central Asian countries like Kazakhstan. Russia is another centre, but it is not
counted in central Asia.

Statement 3: Cultural Centres overseas are ICCR’s important arms, which play significant role in
promotion of India’s soft power strength through its variety of activities like facilitating teaching of
Indian dance, music, yoga, Hindi etc., organizing cultural performances etc.

You can find the list of all such centres at the Q Source.

Q Source: YSK (Release ID :169048)

www.insightsias.com 51
© Insights Active Learning | All rights reserved - 78168. You may not reproduce, distribute or exploit the contents in any form without
written permission by copyright owner. Copyright infringers may face civil and criminal liability
Total Marks : 200
TEST - 3(MOCK TEST - 3) Mark Scored : 89.33

71 The Amasr Act, 2010 that was recently seen in news concerns

A. Preservation of monuments
B. Immigration and refugees in India
C. Genetic cleansing of livestock population
D. Prohibited chemicals and weapons

Your Answer : A
Correct Answer : A

Answer Justification :

Learning: The proposal for amendment of the Amasr Act, 2010 has been approved by the Cabinet
and the pertaining Bill has been moved to Lok Sabha.

The proposal is to allow public works or projects essential for public in prohibited area within 100
meter from protected monument but not having substantial impact on preservation,
2 77 safety, security
3 2
or access to the monument or its immediate surrounding including visual ambiance.
5 0
9
Q Source: Sanjay Kumar/jk/Parl. No. 1/26-07-2017 (Release ID :169062) 41
-
. com
a
72 Private Entrepreneurs Guarantee (PEG) Scheme concerns i l
gm
@
A. Manufacturing MSMEs
s r a7
B. i
Warehousing capacity for food storage
m
t
C.
lkihave
Venture Capital Funds that
u
been established by the Government
D. p
Guaranteed Land acquisition
-
for mega projects
a
Your Answer : B
M isr
it : B
Correct Answer
lk
P u
Answer Justification :

Learning: Under PEG, storage capacity is created by private parties, Central Warehousing
Corporation (CWC) and State Agencies for guaranteed hiring by FCI.

This is because the state alone cannot provide such facilities at all locations. It makes economic
sense for private players to finance the construction of these warehouses and then lease it out for
rent.

Q Source: www.dfpd.nic.in/peg.htm

73 Which of these nations share closest latitudes?


#40166

A. Aswan, Riyadh and Abu Dhabi


B. Cairo, Istanbul and Damascus

www.insightsias.com 52
© Insights Active Learning | All rights reserved - 78168. You may not reproduce, distribute or exploit the contents in any form without
written permission by copyright owner. Copyright infringers may face civil and criminal liability
Total Marks : 200
TEST - 3(MOCK TEST - 3) Mark Scored : 89.33

C. Tehran, Ashgabat and Aleppo


D. Mosul, Muscat and Tel-Aviv

Your Answer : C
Correct Answer : A

Answer Justification :

Justification: Closest latitudes mean that they are located in a narrow band of latitude. They can
be located at different longitudes.

7
2 27
0 3
5
41
-9
. com
ai l
gm
7 @
isra
i t m
p ulk
a -
i sr
M
l k it
Pu

Q Source: Map based questions: West Asia

74 Consider the following with reference to rubber production in India.


1. Kerala is the largest producer of natural rubber in India.
2. The government has deregulated synthetic rubber industry in India.
3. As of date, no dedicated scheme or policy has been enacted to develop rubber sector in India.

Select the correct answer using the codes below.


A. 1 only
B. 2 and 3 only
C. 1 and 2 only
D. 1, 2 and 3

www.insightsias.com 53
© Insights Active Learning | All rights reserved - 78168. You may not reproduce, distribute or exploit the contents in any form without
written permission by copyright owner. Copyright infringers may face civil and criminal liability
Total Marks : 200
TEST - 3(MOCK TEST - 3) Mark Scored : 89.33

Your Answer : A
Correct Answer : C

Answer Justification :

Justification: Statement 1: India is the third largest natural rubber producing country of the world,
next to Thailand and Indonesia, producing about 9 per cent of the global output.

Kerala is the largest producer, followed by TN.

Statement 2: So far as artificial/synthetic rubber is concerned, Chemical & Petrochemical Industry


is delicensed and decontrolled and Government acts as facilitator in this sector.

Statement 3: In order to increase production of Natural Rubber in the country the Government
through the Rubber Board is implementing the scheme “Sustainable and Inclusive Development of
Natural Rubber Sector” wherein support is provided for plantation development and extension,
strengthening research, technology upgradation and market development, Human7resource
development etc. 27 2
3
Q Source: MJPS (Release ID :169314) 1 50
4
-9
m
l .coof major activities in the history of
75 Consider the following matches of periods with the beginning
i
human civilization:
m a
1. Building of megaliths: 3000 years ago
7 @g
2. Cotton cultivation: 2000 years ago
is ra
m ago
3. Domestication of animals: 12000 years
i t
Select the correct answer p ulk
using the codes below.
A. 1 and 2 only ra
-
M
B. 1 and 3 only is
l
C. 2 onlyk it
D. P1,u2 and 3

Your Answer : C
Correct Answer : B

Answer Justification :

Justification: Statement 1: Beginning of cities started about 4700 years ago. Settlement at
Inamgaon began between 3600 and 2700 years ago.

Statement 2: This started at Mehrgarh about 7000 years ago.

Statement 3: Domestication was a gradual process that took place in many parts of the world.

It began about 12,000 years ago. Some of the earliest plants to be domesticated were wheat
and barley.

www.insightsias.com 54
© Insights Active Learning | All rights reserved - 78168. You may not reproduce, distribute or exploit the contents in any form without
written permission by copyright owner. Copyright infringers may face civil and criminal liability
Total Marks : 200
TEST - 3(MOCK TEST - 3) Mark Scored : 89.33

The earliest domesticated animals include sheep and goat.

Q Source: General: Chapter 1: Themes in Indian History: Part -I

76 Which of these strive to ensure equity among Indian citizens?


1. Constitution of India
2. Laws enacted by the Parliament
3. Schemes and programmes of the government

Select the correct answer using the codes below.


A. 1 only
B. 2 and 3 only
C. 1 and 3 only
D. 1, 2 and 3
7
Your Answer : D 2 27
Correct Answer : D 0 3
5
41
Answer Justification : -9
. com
Justification: Statement 1: Fundamental rights ensure l
ai right to equality and remove arbitrary
discrimination. DPSP strives to create a equitable m
g society through special provisions for weaker
@
sections.
s r a7
m i act, Forest Rights Act ensure justice to weaker and
t
Statement 2: Laws like SC/ST Atrocities
marginalized sections. u lki
- p
sra Food security Act etc. ensure that left behind sections are provided with
Statement 3: MGNREGA,
i
livelihood, foodM
etc.
it
u lkPage 10: Social and Political Life: 7 NCERT
P
th
Q Source:

77 The first state in India to introduce the Mid Day Meal (MDM) scheme was

A. Kerala
B. Gujarat
C. Tamilnadu
D. Bihar

Your Answer : D
Correct Answer : C

Answer Justification :

Learning: Other than the TN model of MDM, in 2001, the Supreme Court asked all state
governments to begin this programme in their schools within six months.

www.insightsias.com 55
© Insights Active Learning | All rights reserved - 78168. You may not reproduce, distribute or exploit the contents in any form without
written permission by copyright owner. Copyright infringers may face civil and criminal liability
Total Marks : 200
TEST - 3(MOCK TEST - 3) Mark Scored : 89.33

Later, it was made a central programme that today feeds more than 12 crore children all over
India.

The Midday Meal Scheme is covered by the National Food Security Act, 2013.

The central and state governments share the cost of the Midday Meal Scheme, with the centre
providing 60 percent and the states 40 percent

Q Source: Page 11: Social and Political Life: 7th NCERT

78 The United Nations Children's Fund (UNICEF) is a United Nations (UN) programme that provides
humanitarian and developmental assistance to children and mothers in developing countries.
Consider the following with reference to UNICEF’s association with India.
1. UNICEF provided equipment and technical assistance for the first Penicillin Plant 7
established in
2 7
India.
0 32
2. Within a decade of independence, UNICEF signed an agreement with the
1 5 Government of India to
94
fund some cooperative milk processing plants to support white revolution.
-
3. UNICEF helped India in developing the India Mark II which is now the world’s most widely used
m
handpump. .co
il
m a
Select the correct answer using the codes below. g
A. 1 only a 7@
B. 2 only is r
it m
C. 3 only
l k
D. 1, 2 and 3
- pu
Your Answer : i sra
it M: D
Correct Answer
k
P ul
Answer Justification :

Justification: Statement 3: During the 1970s, UNICEF became a key partner with the Government
of India in the world’s largest rural water supply programme.

UNICEF brought drilling rigs to India which could drill boreholes in hard rock. The Government
supplied handpumps. But, due to some problems, India mark II was later adopted.

Learning: Gender issues were mainstreamed into the training and communication strategy for the
2011 Census with the help of UNESCO.

The Government, in partnership with UNICEF, WHO, the Bill & Melinda Gates Foundation,
Rotary International and the Centers for Disease Control and Prevention contributed to
almost universal awareness of the need to vaccinate all children under five against polio.

www.insightsias.com 56
© Insights Active Learning | All rights reserved - 78168. You may not reproduce, distribute or exploit the contents in any form without
written permission by copyright owner. Copyright infringers may face civil and criminal liability
Total Marks : 200
TEST - 3(MOCK TEST - 3) Mark Scored : 89.33

As a result of these efforts, India was removed from the list of endemic countries in 2014.

Q Source: Additional Research: Page 25: Social and Political Life: 7th NCERT
http://unicef.in/WhoWeAre/History

79 Archaeological Survey of India was founded by

A. Alexander Cunningham
B. Joseph Barruel
C. Lord Lawrence
D. Mortimer Wheeler

Your Answer : A
Correct Answer : A
7
Answer Justification : 2 27
3
50
41 for archaeological research
Learning: ASI is attached to the Ministry of Culture that is responsible
9
and the conservation and preservation of cultural monuments in-the country.
m
co
il. monuments which lasted for over half a
Cunningham carried out a detailed survey of the Buddhist
century. m a
7 @g
sra himself, in the long run, he realised the need for a
While he funded many of his early excavations
i
t m
permanent body to oversee archaeological
i excavations

p ulkPage 6: Themes in Indian History – Part I


Q Source: Additional Research:
a -
isr
M
80 Which of theseitsovereign states do NOT have armed forces?
u lk
1. Costa Rica
P
2. Solomon Islands
3. Madagascar

Select the correct answer using the codes below.


A. 1 and 2 only
B. 3 only
C. 1 and 3 only
D. 2 only

Your Answer :
Correct Answer : A

Answer Justification :

Justification: There are 23 countries that have no active military force, including Costa Rica,
Iceland, Panama, Micronesia, the Marshall Islands, and The Vatican. These nations vary in size,

www.insightsias.com 57
© Insights Active Learning | All rights reserved - 78168. You may not reproduce, distribute or exploit the contents in any form without
written permission by copyright owner. Copyright infringers may face civil and criminal liability
Total Marks : 200
TEST - 3(MOCK TEST - 3) Mark Scored : 89.33

history, and reasons behind choosing to not have a standing army.

Statement 1: Not having an army helped the Costa Rican government to spend the money that the
army would have used, on health, education and other basic needs of the people.

Q Source: Additional Research: Page 28: Social and Political Life: 7th NCERT

81 That the State should “raise the level of nutrition and the standard of living and to improve public
health” is a provision under

A. Fundamental right
B. Directive Principle of State policy
C. Fundamental Duty
D. Legal rights

Your Answer : B 7
Correct Answer : B
2 27
0 3
5
Answer Justification : 41
-9
. com
Learning: Right to Health is not included as an explicit fundamental right in the Indian
Constitution. ial
m
@g
Most provisions related to health are under DPSP.
7
is ra
These are:
k i tm
p ul
a
Article 38 says that- the state will secure a social order for the promotion of welfare of the
r
is affordable healthcare is one of the ways to promote welfare.
people. Providing
M
l k it
u
PArticle 39(e) calls the state to make sure that health and strength of workers, men and
women, and the tender age of children are not abused.

Article 41 imposes duty on state to provide public assistance in cases of unemployment, old
age, sickness and disablement etc.

Article 42 makes provision to protect the health of infant and mother by maternity benefit.

Article 47 make it duty of the state to improve public health, securing of justice, human
condition of works, extension of sickness, old age, disablement and maternity benefits and
also contemplated. Further, State’s duty includes prohibition of consumption of intoxicating
drinking and drugs are injurious to health.

Article 48A ensures that State shall Endeavour to protect and impose the pollution free

www.insightsias.com 58
© Insights Active Learning | All rights reserved - 78168. You may not reproduce, distribute or exploit the contents in any form without
written permission by copyright owner. Copyright infringers may face civil and criminal liability
Total Marks : 200
TEST - 3(MOCK TEST - 3) Mark Scored : 89.33

environment for good health.

Q Source: Page 29: Social and Political Life: 7th NCERT

82 Which of these nations has nationalized the Suez Canal?

A. Egypt
B. Syria
C. Israel
D. Saudi Arabia

Your Answer : A
Correct Answer : A

Answer Justification : 7
2 27
Learning: The Suez Canal is considered to be the shortest link between the
5 03east and the west due
to its unique geographic location.
9 41
-
m
o the Mediterranean sea at Port said
It is an important international navigation canal linking between
l . c
and the red sea at Suez.
m ai
Egypt nationalized the canal in 1956.
7 @g
isra
k i tm
p ul
a -
isr
M
l k it
Pu

Q Source: Map based questions: West Asia

83 Consider the following statements.


1. A Governor is the Head of a State of India, whereas President is the head of the Indian state.
2. Governors are appointed by the President of India and he is free to consult the state chief minister
in this matter.

www.insightsias.com 59
© Insights Active Learning | All rights reserved - 78168. You may not reproduce, distribute or exploit the contents in any form without
written permission by copyright owner. Copyright infringers may face civil and criminal liability
Total Marks : 200
TEST - 3(MOCK TEST - 3) Mark Scored : 89.33

Which of the above is/are correct?


A. 1 only
B. 2 only
C. Both 1 and 2
D. None

Your Answer : C
Correct Answer : C

Answer Justification :

Justification: Statement 1: The Head (ceremonial) of the State is the Governor. Head of
government is the Chief Minister.

She/He is appointed by the Central Government to ensure that the State Government works within
the rules and regulations of the Constitution.
77
22 to do so since
Statement 2: The President may nor may not consult the CM, but it is a convention
3
both need to work in tandem. 50 1
4
Q Source: Page 33: Social and Political Life: 7 NCERT th
-9
. com
ai l
g m
84 An Internal Complaints Committee (ICC) is usually constituted at workplaces according to which of
these acts? @
s r a7
m i
t
A. Consumer Protection Act, 1984
B. Factories Act, 1948
u lki
p Women at Workplace (Prevention, Prohibition and Redressal) Act,
C. Sexual Harassment- of
2013
i sra
k it M above
D. None of the

Your P
ul
Answer : C
Correct Answer : C

Answer Justification :

Learning: Under the Act, which also covers students in schools and colleges as well as patients in
hospitals, employers and local authorities will have to set up grievance committees to investigate all
complaints.

These grievance committees are known as ICC.

The District Officer is required to constitute a Local Complaints Committee at each district,
and if required at the block level.

The Complaints Committees have the powers of civil courts for gathering evidence.

www.insightsias.com 60
© Insights Active Learning | All rights reserved - 78168. You may not reproduce, distribute or exploit the contents in any form without
written permission by copyright owner. Copyright infringers may face civil and criminal liability
Total Marks : 200
TEST - 3(MOCK TEST - 3) Mark Scored : 89.33

Q Source: Additional Research: Page 64: Social and Political Life: 7th NCERT

85 The Advertising Standards Council of India (ASCI) is a


1. Self-regulatory institution
2. Non-government body
3. Body that can prohibit certain ads from appearing on TV

Select the correct answer using the codes below.


A. 1 and 2 only
B. 3 only
C. 2 only
D. 1, 2 and 3

Your Answer : D
Correct Answer : D
7
2 27
Answer Justification :
3
1 50
4consumer is his audience.
-9
Concept & Background: When an advertiser is creating an ad, the

. com
l
ai is in bad taste or is false in its claims, they
If a consumer feels that a particular advertisement
need a body or council to whom they can air m
g their grievances and who will take any
@
appropriate action, if necessary.
s r a7
m i
i t
ASCI as a self-regulatorylk
body governing advertising content is the ideal medium as its
u
p the advertisers as well as the consumers.
purpose is to serve- both

i sra
Mother non governmental body in India which regulates the advertising content that
t no
There iis
l k
Pisureleased in India.
Justification: If an ad that is released in India seems objectionable, a person can write to ASCI
with their complaint.

This complaint will be deliberated on by the complains council after providing due process to
advertiser to defend the ad against the complaint. If complaint is upheld then the ad is voluntarily
either withdrawn or modified.

In 2007, the Government of India amended the Cable TV Network Rules’ Advertising Code by which
ads which violate ASCI code cannot be permitted on TV.

Q Source: Additional Research: Page 83: Social and Political Life: 7th NCERT

86 Which of these nations border both Caspian Sea and Black Sea?

www.insightsias.com 61
© Insights Active Learning | All rights reserved - 78168. You may not reproduce, distribute or exploit the contents in any form without
written permission by copyright owner. Copyright infringers may face civil and criminal liability
Total Marks : 200
TEST - 3(MOCK TEST - 3) Mark Scored : 89.33

A. Iran
B. Turkey
C. Russia
D. Georgia

Your Answer : C
Correct Answer : C

Answer Justification :

Learning:

7
2 27
0 3
5
41
-9
. com
ai l
gm
7 @
isra
i t m
p
Q Source: Map based questions: ulk
West Asia
a -
isr
k it M statements.
87 Consider the following
ul in India has a Legislative Assembly.
1. Every state
P
2. The strength of legislative assembly in a state is fixed by the President.

Which of the above is/are correct?


A. 1 only
B. 2 only
C. Both 1 and 2
D. None

Your Answer : D
Correct Answer : A

Answer Justification :

Justification: Statement 1: But, all UTs do not have a legislative assembly. Only, Delhi and
Puducherry have it.

Statement 2: This is based on the population of the state. A state with a larger population like UP
www.insightsias.com 62
© Insights Active Learning | All rights reserved - 78168. You may not reproduce, distribute or exploit the contents in any form without
written permission by copyright owner. Copyright infringers may face civil and criminal liability
Total Marks : 200
TEST - 3(MOCK TEST - 3) Mark Scored : 89.33

will have more seats in the assembly.

Q Source: Page 32: Social and Political Life: 7th NCERT

88 Nations that border the Persian Gulf are


1. Saudi Arabia
2. United Arab Emirates
3. Iran
4. Kuwait
5. Jordan

Select the correct answer using the codes below.


A. 2 and 4 only
B. 1, 2, 3 and 4 only
C. 3 and 5 only
D. 1 and 5 only 7
2 27
0 3
Your Answer : B 5
41
-9
Correct Answer : B

Answer Justification : . com


ai l
Justification: gm
7 @
isra
i t m
p ulk
a -
i sr
M
l k it
Pu

Q Source: Map based questions: West Asia

89 What are the properties of X-rays?


1. X-rays have much longer wavelengths than visible light.
2. X-ray photons carry enough energy to ionize atoms and disrupt molecular bonds.
3. X-rays do not interact with matter at all.

www.insightsias.com 63
© Insights Active Learning | All rights reserved - 78168. You may not reproduce, distribute or exploit the contents in any form without
written permission by copyright owner. Copyright infringers may face civil and criminal liability
Total Marks : 200
TEST - 3(MOCK TEST - 3) Mark Scored : 89.33

Select the correct answer using the codes below.


A. 1 and 2 only
B. 2 only
C. 3 only
D. 1, 2 and 3

Your Answer : B
Correct Answer : B

Answer Justification :

Justification: Statement 1: They have shorter wavelengths and higher frequencies.

This makes it possible to probe structures much smaller than can be seen using a normal
microscope.
7
2 27
3
This property is used in X-ray microscopy to acquire high resolution0images, and also in X-ray
5
crystallography to determine the positions of atoms in crystals.
9 41
-
m
.cotherefore harmful to living tissue.
Statement 2: This makes it a type of ionizing radiation, land
i
m a
@ g
a
A very high radiation dose over a short7 period of time causes radiation sickness, while lower
s r
doses can give an increased riski of radiation-induced cancer.
k i tm
p ul
- with matter in three main ways, through photoabsorption, Compton
Statement 3: X-rays interact
a
i r
s scattering.
scattering, and Rayleigh

k itM
ul strength of these interactions depends on the energy of the X-rays and the elemental
PThe
composition of the material.

We will try covering these separately in upcoming tests because they are important.

Q Source: Additional Research: Page 25: Social and Political Life: 7th NCERT

90 Consider the following statements.


1. Dasht-e Kavir is a great salt desert located in Arabian Peninsula.
2. Dasht-e Lut is a large desert located in Iran that has been inscribed on UNESCO's World Heritage
List.

Which of the above is/are correct?


A. 1 only
B. 2 only
C. Both 1 and 2
www.insightsias.com 64
© Insights Active Learning | All rights reserved - 78168. You may not reproduce, distribute or exploit the contents in any form without
written permission by copyright owner. Copyright infringers may face civil and criminal liability
Total Marks : 200
TEST - 3(MOCK TEST - 3) Mark Scored : 89.33

D. None

Your Answer :
Correct Answer : B

Answer Justification :

Justification: Statement 1: Also known as Kavir-e Namak (literally 'salty lowlands') and the Great
Salt Desert, it is a large desert lying in the middle of the Iranian plateau.

Statement 2: The Lut Desert, widely referred to as Dasht-e Lut is a large salt desert located in the
provinces of Kerman and Sistan and Baluchestan, Iran.

It is the world's 27th-largest desert, and was inscribed on UNESCO's World Heritage List in
2016.
7
2 27
The surface of its sand has been measured at temperatures as high as370 °C making it one of
the world's driest and hottest places. 1 50
4
-9
. com
ai l
m
7 @g
isra
k i tm
p ul
a -
is r
M
l k it
Pu

Q Source: Map based questions: West Asia

www.insightsias.com 65
© Insights Active Learning | All rights reserved - 78168. You may not reproduce, distribute or exploit the contents in any form without
written permission by copyright owner. Copyright infringers may face civil and criminal liability
Total Marks : 200
TEST - 3(MOCK TEST - 3) Mark Scored : 89.33

91 Harappans were aware of and practiced which of these number systems in their weights and
measures?

A. Binary system
B. Decimal system
C. Hexadecimal system
D. Both (a) and (b)

Your Answer :
Correct Answer : D

Answer Justification :

Learning: Exchanges were regulated by a precise system of weights, usually made of a stone called
chert and generally cubical with no markings.

77higher
The lower denominations of weights were binary (1, 2, 4, 8, 16, 32, etc). While the
2
2
3 used for weighing
denominations followed the decimal system, the smaller weights were probably
jewellery and beads and bigger weights were used for food grains. 150
- 94
m –I
Q Source: Additional Research: Page 15: Themes in Indian History
o
a il.c
gmAhar or Banas culture was discovered in the
92 Among the earliest chalcolithic cultures in India, the
a 7@
A. Mewar region of Rajasthan is r
it
B. Gwalior region of Madhya Pradesh
m
l k
C. Bundelkhand region p ofuUttar Pradesh
-
sra
D. Kangra region of Himachal Pradesh

M i
Your Answerit: A
l k
CorrectuAnswer :A
P
Answer Justification :

Learning: Nearly one hundred sites of the culture have been located along its principal axis, i.e.,
the valleys of river Banas and its tributaries and subtributaries in Banswara, Udaipur etc.

The technology at Ahar was based mainly on copper and very few microblades and microliths have
been discovered.

Ahars were a separate culture from GJs. Located in northeast Rajasthan, the Ganeshwar-Jodhpura
complex which was an early centre of agriculture and copper metallurgy in the subcontinent.

www.insightsias.com 66
© Insights Active Learning | All rights reserved - 78168. You may not reproduce, distribute or exploit the contents in any form without
written permission by copyright owner. Copyright infringers may face civil and criminal liability
Total Marks : 200
TEST - 3(MOCK TEST - 3) Mark Scored : 89.33

7
2 27
Q Source: Additional Research: Page 12: Themes in Indian History – I
0 3
5
9 41
93 Consider the following statements. -
m
l
1. Most Ashokan inscriptions were in the Greek language while
i .co those in the northwest of India
subcontinent were in Aramaic and Prakrit.
m a
@g and Brahmi scripts.
2. Ashokan inscriptions were written in both Prakrit
7
is ra
Which of the above is/are correct? m
A. 1 only l k it
B. 2 only
- pu
C. Both 1 and 2 ra
D. None M is
l k it
Your P u :D
Answer
Correct Answer : B

Answer Justification :

Justification: Statement 1: A large number of them were in Prakrit, but in the NW direction one
could find Aramaic and Greek languages.

Statement 2: The Aramaic and Greek scripts were used for inscriptions in Afghanistan.

James Prinsep, an officer in the mint of the East India Company, deciphered Brahmi and Kharosthi,
two scripts used in the earliest inscriptions and coins. This gave a new direction to investigations
into early Indian political history.

Q Source: Page 28: Themes in Indian History – I

94 With reference to Matsays, one of the early states, consider the following statements.

www.insightsias.com 67
© Insights Active Learning | All rights reserved - 78168. You may not reproduce, distribute or exploit the contents in any form without
written permission by copyright owner. Copyright infringers may face civil and criminal liability
Total Marks : 200
TEST - 3(MOCK TEST - 3) Mark Scored : 89.33

1. They were one of the Indo-Aryan tribes of Vedic India.


2. Their rule extended to the whole of eastern and central India.
3. They were an appendage of Ashmaka dynasty.

Select the correct answer using the codes below.


A. 1 only
B. 2 and 3 only
C. 3 only
D. 1, 2 and 3

Your Answer :
Correct Answer : A

Answer Justification :

Justification: Statement 1 and 2: By the late Vedic period, they ruled a kingdom located south of
the Kurus, and west of the Yamuna River which separated it from the kingdom of7the7 Panchalas.
2 2
It roughly corresponded to the former state of Jaipur in Rajasthan. 5 03
1
94
- Bairat). Alwar has been a part
Statement 3: The capital of Matsya was at Viratanagari (present-day
of Matsya region. Ahmakas were further down south, seecmap
m
o below.
il .
m a
@ g
7
isra
i t m
p ulk
a -
i sr
M
l k it
Pu

www.insightsias.com 68
© Insights Active Learning | All rights reserved - 78168. You may not reproduce, distribute or exploit the contents in any form without
written permission by copyright owner. Copyright infringers may face civil and criminal liability
Total Marks : 200
TEST - 3(MOCK TEST - 3) Mark Scored : 89.33

Q Source: Page 30: Themes in Indian History – I

95 In Ancient India, a means of claiming high status for the Kings was to identify with a variety of
deities. Consider the following with reference to it.
1. Kushana rulers installed god-like colossal statues of them.
2. Some Kushanas adopted the title devaputra which implied the “son of the God”.
3. Vijayanagara kings claimed to rule on behalf of the god Virupaksha.
4. Vijayanagara rulers adopted the title ‘brahma-varna” to segment themselves as the highest castes
equivalent to God.

Select the correct answer using the codes below.


A. 1, 2 and 3 only
B. 2 and 4 only
C. 3 only
D. 1, 2, 3 and 4
7
Your Answer : A 2 27
0 3
Correct Answer : A 5
41
Answer Justification : -9
. com
Justification: Statement 1: Huge statues of Kushanas l
aihave been found installed in a shrine at Mat
m
g been found in a shrine in Afghanistan as well.
near Mathura (Uttar Pradesh). Similar statues have
@
s r a7 inspired by Chinese rulers who called themselves
i
Statement 2: The title devaputra was possibly
m
t
lki
sons of heaven.
u
p their close links with the gods by using the title “Hindu Suratrana”
-
Statement 3: They indicated
i sra
meaning Hindu Sultan.

k it M Research: Page 36: Themes in Indian History – I


Q Source:l Additional
Pu
96 The institution of Samanta finds mention for the first time in epigraphs of northern India dating to
the 6th century. It is most closely associated with the origin and growth of which of these systems in
India?

A. Feudalism
B. Bonded Labour
C. State ownership of land
D. Standing army system

Your Answer : B
Correct Answer : A

Answer Justification :

Justification: Options B, C and D existed much prior to 6th century, so they can’t be the answer.

www.insightsias.com 69
© Insights Active Learning | All rights reserved - 78168. You may not reproduce, distribute or exploit the contents in any form without
written permission by copyright owner. Copyright infringers may face civil and criminal liability
Total Marks : 200
TEST - 3(MOCK TEST - 3) Mark Scored : 89.33

The institution is considered to belong properly to the Gupta Empire, however, some vague
evidence is found in South India as well.

The Samanta in South-India was used to mean a vassal to an emperor. In North-India, the
earliest use of the term in a similar sense was in Bengal where someone was described as
Samanta-Chudamanih (best among feudatories) of the imperial Guptas.

The Samanta vassal provided military support to the Monarch and governed over a portion of
a territory.

Learning: The term Indian feudalism is used to describe zamindar, jagirdar, sardar, mankari,
deshmukh, chaudhary and samanta.

7
In this system, a nobility gave the emperor military services in exchange of7land.
2 2
5 03
1
There was a separate class of labourers or peasants who were4provided military protection in
9
exchange of labour and share of their produce. They were-allowed to live on the lord’s land.

. com
ai l
Q Source: Additional Research: Themes in Indianm
History – I
@ g
ra7
isto bear the names and images of rulers were issued by the
97 In the Indian subcontinent, the first coins
m
l k it
u
A. Indo - Greeks
-p
B. Mauryas
i sra
C. ParthiansM
D. kit
Yaudheyas
l
Pu
Your Answer : A
Correct Answer : A

Answer Justification :

Learning: Mauryas issued punch marked coins made of silver and copper.

Indo-greeks used names of rulers on coins. First gold coins were issued by Kushanas (some
sources dispute this fact).

Coins were also issued by tribal republics of Punjab and Haryana called the Yaudheyas.

Archaeologists have unearthed several thousand copper coins issued by the Yaudheyas.

www.insightsias.com 70
© Insights Active Learning | All rights reserved - 78168. You may not reproduce, distribute or exploit the contents in any form without
written permission by copyright owner. Copyright infringers may face civil and criminal liability
Total Marks : 200
TEST - 3(MOCK TEST - 3) Mark Scored : 89.33

Some of the most spectacular gold coins were issued by the Gupta rulers.

Q Source: Page 44: Themes in Indian History – I

98 The earliest inscriptions recording royal land grants were issued during

A. Guptas
B. Satvahanas
C. Pallavas
D. Hoysalas

Your Answer : A
Correct Answer : B

Answer Justification : 7
2 27
Learning: In early India, land possessed great value and the gift of an estate
5 03 was a marker of
status.
9 41
-
o m
i
While the earliest inscriptions recording royal landl .cgrants were issued during Satavahana
a
m fourth century CE.
rule, the practice truly grew in scope from the
g
a 7@
s r
By the 5th-6th centuries, ruling idynasties across the subcontinent, as well as their
t m
lki were all engaged in making such grants.
subordinates and feudatories
u
- p
i sra were usually engraved on sheets of hammered copper. They are
Land grant charters
it Mas tamrapatta, a ‘plate of copper’ or tamrashasana, ‘an order in copper’.
described
k
l
Pu
Q Source: Page 50: Themes in Indian History – I

99 What is a catcher beach?

A. A beach with low shorelines


B. A tidal estuary with no opening mouth
C. A place where marine debris tends to pile up or aggregate
D. A beach devoid of mangroves and corals

Your Answer :
Correct Answer : C

Answer Justification :

Learning: A catcher beach should not be confused with a dumping ground or heavily trashed public

www.insightsias.com 71
© Insights Active Learning | All rights reserved - 78168. You may not reproduce, distribute or exploit the contents in any form without
written permission by copyright owner. Copyright infringers may face civil and criminal liability
Total Marks : 200
TEST - 3(MOCK TEST - 3) Mark Scored : 89.33

beach.

A catcher beach typically receives its accumulations of debris due to its shape and location in
combination with high-energy waves, storms, or winds.

A specific example of a catcher beach can be found along the shores of Gore Point, Alaska. The
geography of this location makes it a very high-density catcher beach, as it sticks out like a hook
into the Gulf of Alaska current.

Q Source: Surprise questions

100 The Centre has banned the manufacture and import of Polychlorinated Biphenyls (PCBs). This is
because

A. It is a toxic pollutant.
B. It is responsible for coral bleaches across the eastern coast of India.
C. It was being oversupplied in the market leading to price crashes and poor 77
2producer margins.
2
D. 03
It was heavily used by farmers as an additive in their NPK fertilizers.
5
1
Your Answer : A - 94
Correct Answer : A
. com
a i l
Answer Justification : m
7 @g
Learning: PCBs are synthetic organic s
i ra
chemicals used in many different products including
tm flame-retardants, and paints.
electrical equipment, inks, adhesives,
i
lk
- pu
a use are now severely restricted in many countries because of possible
Its production rand
s
i
impacts on human health and the environment.
M
l k it
u
PThe use of PCBs in any form shall be completely prohibited by 2025.

The government has directed users not to drain or discharge PCBs directly or indirectly on
land, in surface water or from effluent treatment plants.

The government's decision is in line with the Stockholm Convention, under which signatory
countries are to prohibit or take necessary legal and administrative measures to eliminate the
production and use of PCBs.

Q Source:
http://timesofindia.indiatimes.com/city/delhi/Govt-bans-manufacture-import-of-toxic-pollutant-PCBs/
articleshow/51793188.cms

www.insightsias.com 72
© Insights Active Learning | All rights reserved - 78168. You may not reproduce, distribute or exploit the contents in any form without
written permission by copyright owner. Copyright infringers may face civil and criminal liability

You might also like